General Dermatology Flashcards
1- A 32-year-old patient from Iraq presents with a new oral ulceration. He has had several oral ulcerations before that resolved on their own. Upon further physical examination, you notice he also has redness of both eyes and tender nodules on both shins. Cultures have not grown any organisms to date. A biopsy shows lymphocytic vasculitis with a neutrophilic angiocentric infiltrate. Which of the following is the most likely diagnosis?
A. Sweet’s syndrome
B. Familial Mediterranean fever
C. Pyoderma gangrenosum
D. Lofgren’s syndrome
E. Behcet’s disease
Correct choice: E. Behcet’s disease
Explanation: Behcet’s disease is a multisystem polysymptomatic disease with cutaneous findings ranging from sterile papulopustules and palpable purpura to erythema nodosum-like lesions. The diagnosis is based on International Study Group criteria that includes: recurrent oral ulceration, recurrent genital ulceration, ocular abnormalities (uveitis, retinal vasculitis), and skin lesions. On a skin biopsy, the characteristic finding is a neutrophilic angiocentric infiltrate with leukocytoclastic (early) or lymphocytic (late) vasculitis.
1 – Sweet’s syndrome is histologically characterized by superficial dermal edema, diffuse dermal neutrophils and traditionally no true vasculitis (no vessel wall necrosis), but nuclear dust is common. 2 – Familial Mediterranean fever is an autosomal recessive inherited disorder characterized by recurrent febrile episodes lasting for 1-2 days, with painful self-limiting episodes of pleuritis, peritonitis or synovitis. Cutaneous manifestations include erysipelas-like erythema over the joints, lower legs and dorsal feet in addition to small vessel vasculitis and nonspecific purpura. 3 – Pyoderma gangrenosum is characterized by a necrotic epidermis and ulceration, occasionally with pustules and sometimes pseudoepitheliomatous hyperplasia at the edge of the ulcer. It also has a diffuse infiltrate of neutrophils, lymphocytes, and histiocytes in the dermis, and occasionally has vasculitis. 4 – Lofgren’s syndrome is a more acute yet transient form of sarcoidosis with erythema nodosum, hilar adenopathy, fever, polyarthritis and iritis. Sarcoidosis is characterized by non- caseating, well-demarcated granulomas in the dermis or subcutaneous tissue.
2- A 42-year-old male presents with multiple round to oval hypomelanotic patches measuring several centimeters in size. The lesions coalesce centrally and are more prominent on the trunk. Associated fine scale is apparent upon scratching the skin. The distribution is symmetric and favors the chest, abdomen and back. Which of the following is not a proposed mechanism of the hypo pigmented skin in tinea versicolor?
A. Azelaic acid inhibits tyrosinase produced by Malassezia furfur
B. Abnormal melanosome production
C. Decreased melanin synthesis
D. Partial block in melanosome transfer to keratinocytes
E. Kojic acid plays a role in decreased melanin synthesis
Correct choice: E. Kojic acid plays a role in decreased melanin synthesis
Explanation: Kojic acid does not play a role in decreased melanin synthesis. In the hypopigmented skin of tinea versicolor, there is a decreased density of melanosomes within keratinocytes, but no change in the melanocyte density. Abnormal melanosome production, decreased melanin synthesis, and a partial block in melanosome transfer to keratinocytes have all been suggested as underlying defects. Azelaic acid, a competitive inhibitor of tyrosinase produced by Malassezia furfur, plays a role in the decreased melanin synthesis. Although the differential diagnosis may include postinflammatory hypopigmentation (e.g. secondary to parapsoriasis), progressive macular hypomelanosis, and early vitiligo, the diagnosis of tinea versicolor is easily confirmed by examination of the associated scale in a potassium hydroxide preparation.
1 - Azelaic acid inhibits tyrosinase produced by Malassezia furfur: Azelaic acid is a competitive inhibitor of tyrosinaes produced by Malassezia furfur and plays a role in decreased melanin synthesis. 2 - Abnormal melanosome production: Abnormal melanosome production, decreased melanin synthesis and a partial block in melanosome transfer to keratinocytes have all been suggested as underlying defects in tinea versicolor.
3 - Decreased melanin synthesis: Abnormal melanosome production, decreased melanin synthesis and a partial block in melanosome transfer to keratinocytes have all been suggested as underlying defects in tinea versicolor.
4 - Partial block in melanosome transfer to keratinocytes: Abnormal melanosome production, decreased melanin synthesis and a partial block in melanosome transfer to keratinocytes have all been suggested as underlying defects in tinea versicolor.
3- Which of the following is TRUE?
A. Malignant transformation occurs in 2-13%
B. These are pathognomonic of Von Recklinghausen disease
C. They are composed of a mixture of the neuromesenchyme (Schwann cells, endoneurial fibroblasts, perineurial cells)
D. Relative ratio of axons to Schwann cells in these tumors is always >1:1
E. These commonly display overlying hypertrichosis or hyperpigmentation
Correct choice: C. They are composed of a mixture of the neuromesenchyme (Schwann cells, endoneurial fibroblasts, perineurial cells)
Explanation: C is correct and the photograph displays a neurofibroma. A, B, D, & E all describe plexiform neurofibromas. For answer D, the ratio of axons to Schwann cells is always <1:1, since axons do not replicate but Schwann cells can.
4- A 37 week primigravid female presents with pruritic rash and blisters involving the abdomen and extremities. What is the pathognomonic cell type on H&E?
A. Neutrophils
B. Eosinophils
C. Lymphocytes
D. Mast cells
E. Plasma cells
Correct choice: B. Eosinophils
Explanation: Eosinophils are the predominant cells see in histology of pemphigoid gestationis.
5- A 42-year-old female with keratosis follicularis has a severe disease flare after starting a new medication. She has a history of epilepsy, bipolar disorder and gout. What is the most likely new treatment that induced the flare?
A. Phenytoin
B. Valproate
C. Carbamazepine
D. Allopurinol
E. Lithium
Correct choice: E. Lithium
Explanation: Keratosis follicularis (or Dariers disease) is known to worsen with initiation of certain medications including lithium. The other medications listed are not known to worsen keratosis follicularis.
6- Dihydroxyacetone is found in which of the following products?
A. Rubber accelerators
B. Shampoos
C. Artificial nails
D. Hair dyes
E. Sunless tanning preparations
Correct choice: E. Sunless tanning preparations
Explanation: Dihydroxyacetone is the active ingredient in sunless tanning preparations. Upon oxidation it turns orange-brown and binds to the stratum corneum. Rubber accelerators contain mercaptobenthothiazole, carba mix, thiuram mix, or mercapto mix. Shampoos, especially “tear- free” ones, may contain cocamidopropyl betaine. Artificial nails may contain methyl methacrylate or ethyl acrylate. Hair dyes often have paraphenylenediamine.
7- What is the diagnosis:
A. Alopecia areata
B. Trichotillomania
C. Androgenetic alopecia
D. Tinea capitis
E. Temporal triangular alopecia
Correct choice: A. Alopecia areata
Explanation: Alopecia areata presents with discrete, round or oval patches of non-scarring hair loss. Initial sparing of non-pigmented hair may occur leading patients to report rapid whitening of the hair. Similarly, initial regrowth of hair may include white or grey hairs.
Trichotillomania is a an impulse disorder characteriszed by repetitive hair pulling. As a result, varying lengths of hair can be seen within areas of alopecia. The mainstay of treatment is behavioral modification.
Androgenetic alopecia is an androgen-dependent form of nonscarring alopecia. It affects men more commonly than women and frequency increases with age. The characteristic pattern in men includes frontalparietal recession and thinning of the vertex hair. In women, central thinning is most commonly seen.
Traction alopecia is the result of chronic mechanical trauma to the frontal and bitemporal scalp. Though initially nonscarring, with repetitive traction, hair loss can become permanent.
Temporal triangular alopecia is a form of congenital alopecia. Presenting at birth or in early childhood, there is unilateral or bilateral nonscarring alopecia of the temporal area. The hair loss is permanent.
8- You are evaluating a 44-year-old man who presents with a new rash. He takes off his shirt and raises his arms to show you. In the left axilla, you see a hyperpigmented velvety plaque. Which one of the following medical conditions should consider?
A. Diabetes mellitus
B. Kidney stones
C. Hepatitis B virus infection
D. Hypertension
E. Benign prostatic hyperplasia
Correct choice: A. Diabetes mellitus
Explanation: The patient likely has acanthosis nigricans associated with diabetes mellitus and insulin resistance. Acanthosis nigricans is not associated with kidney stones, hepatitis B virus infection, hypertension, or benign prostatic hyperplasia.
9- A 15-year-old boy presents with keratotic follicular papules involving the bilateral dorsal arms, thighs, and cheeks. Analysis of an unaffected region of skin would likely exhibit which of the following findings:
A. Diminished filaggrin
B. Diminished loricrin
C. Diminished involucrin
D. Diminished lamellar bodies
E. Decreased Transglutaminase I activity
Correct choice: A. Diminished filaggrin
Explanation: Filaggrin mutations are most commonly associated with atopic dermatitis, but have also been implicated in keratosis pilaris as described in this question. Lamellar bodies are diminished in Flegel’s disease and absent in Harlequin icthyosis. Diminished tissue transglutaminase I activity is associated with lamellar icthyosis and non-bullous congenital icthyosiform erythroderma. The other findings are not classically associated with keratosis pilaris.
10- What is the most likely diagnosis?
A. Mixed Cryoglobulinemia
B. Cold Panniculitis
C. Calciphylaxis
D. Homozygous Protein C deficiency
E. Sneddon Syndrome
Correct choice: C. Calciphylaxis
Explanation: An obese patient with dusky, retiform and stellate necrotic plaques predominantly on the fatty areas of the body is most consistent with calciphylaxis. The differential diagnosis includes warfarin necrosis. Oxalate or cholesterol emboli can in rare cases resemble calciphylaxis, but tend to affect distal extremities as opposed to fatty, central areas.
Mixed cryoglobulinemia is a vasculitis, which manifests with palpable purpura as opposed to retiform or stellate purpura and necrosis. Cold panniculitis is a subcutaneous disorder which manifests with painful pink-red to violaceous plaques - this condition is seen most often in newborns, but can affect adults and is usually seen on the outer thighs of adults (equestrian panniculitis). Homozygous deficiency or severe dysfunction of either protein C or protein S leads to neonatal purpura fulminans within a few hours to 5 days after birth, and it is fatal unless treated. Sneddon syndrome is a vascular disorder which manifests with widespread livedo reticularis or racemosa with CNS problems (stroke, TIA, seizures). There is no retiform purpura or necrosis apparent.
11- A 21 year-old Mexican female presents to clinic with these asymptomatic skin findings (see clinical images below) that she has had since she was 2 years old. Her biopsy results are shown in the histopathology image below. Which of the following is true regarding the disorder?
A. The disorder can be inherited in an autosomal dominant fashion, but most cases are sporadic.
B. This condition does not undergo malignant degeneration
C. The characteristic finding on histopathology is a keratin-filled epidermal invagination with an angulated parakeratotic tier
D. Both A and C are correct
D. All of the above are correct
Correct choice: D. Both A and C are correct
Explanation: This patient has linear porokeratosis.
Porokeratosis is a disorder of keratinization characterized by hyperkeratotic plaques surrounded by a peripheral ridge-like scale that expands centrifugally. The diagnosis is typically made clinically but biopsy can help by showing the hallmark cornoid lamella which is a thin column of tightly packed parakeratotic cells extending from an invagination through the adjacent stratum corneum with an attenuated or absent granular layer.1 There are several recognized variants, the most common are porokeratosis of Mibelli and disseminated superficial actinic porokeratosis, with less common variants including linear porokeratosis, and punctate porokeratosis. Porokeratosis can be inherited in an autosomal dominant fashion, but most cases are sporadic.
When managing patients with porokeratosis, it is important to remember there is an increased risk for malignant transformation. Most at risk are those with the linear variant who have up to an 11% lifetime risk of developing skin cancer in a lesion with a latency period of 3-4 decades after initial diagnosis.2 Risk factors include longer duration, larger plaques and acral involvement. The most common cancers are SCC and Bowen’s disease and there may be multiple types in the same plaque.2
12- A patient presents for evaluation of a solitary painful lesion on the shin. She feels well and denies any systemic symptoms. A recent fasting blood glucose was within normal limits. Which of the following is the most likely diagnosis?
A. Pretibial myxedema
B. Morphea
C. Lipodermatosclerosis
D. Necrobiotic xanthogranuloma
E. Necrobiosis lipoidica
Correct choice: E. Necrobiosis lipoidica
Explanation: Concomitant diabetes has been reported in 11-65% of patients with necrobiosis lipoidica (NL).
This question asks the examinee to identify NL (choice E) by a typical yellowish-brown plaque with prominent atrophy and telangiectasia on the shin, recognizing that a history of diabetes is not required to make the diagnosis. Once widely called necrobiosis lipoidica diabeticorum, NL is now the preferred term for this condition, given that 11-65% of patients with NL have diabetes; conversely, 0.3-1.2% of diabetics have NL. Pretibial myxedema (choice 1) presents on the shins with waxy indurated plaques or nodules with a peau d’orange quality. Plaque-type morphea (choice 2) favors the trunk and proximal extremities with erythematous-violaceous patches that progress to indurated, ivory white to hyperpigmented plaques with lilac borders. Lipodermatosclerosis (choice 3) manifests in the acute phase with painful erythematous plaques on the lower legs above the malleoli, and chronically with sclerotic reddish-brown plaques resembling an inverted wine bottle. Necrobiotic xanthogranuloma (NXG, choice 4) shows indurated yellowish papules, plaques, or nodules with variable atrophy, telangiectasia, and ulceration that can cause scarring. While it can morphologically resemble NL, NXG most commonly develops in the periorbital region rather than on the shins.
13- Which of the following hormonal contraceptive methods are most effective in treating acne?
A. intrauterine device (IUD)
B. norgestimate
C. levonorgestrel
D. depot injection
E. drospirenone
Correct choice: E. drospirenone
Explanation: Drospirenone. While some hormonal contraceptive methods are used to treat acne, some have been shown to worsen acne. Combined oral contraceptives (COC) and the vaginal ring improved acne, whereas depot injections, subdermal implants, and hormonal intrauterine devices worsened acne. Within the COC category, drospirenone was the most effective at treating acne.
14- A 59 year old female presents to the dermatology office with the complaint of a red, non-scaly, asymptomatic rash of the lower extremities. She was on a 4-hour walking tour of Boston the day before she noticed the rash. What is the most likely diagnosis?
A. Cutaneous T cell lymphoma
B. Allergic contact dermatitis
C. Lichen planus
D. Psoriasis
E. Pigmented purpuric dermatosis
Correct choice: E. Pigmented purpuric dermatosis
Explanation: The patient has pigmented purpuric dermatosis (PPD) representing with extravasated red blood cells with a sharp demarcation at the sock line on her leg. PPD presents in several forms with the most common form being Schamberg disease with red-to-orange cayenne-pepper macules on the bilateral lower extremities in the setting of dependent edema. Treatment consists of leg elevation, compression stockings, and topical steroids if pruritic. In addition, anti-oxidants by mouth may improve the condition as well.
Cutaneous T cell lymphoma is on the differential diagnosis of PPD but given the acuity of the onset, the history, and the sock line demarcation, PPD is more likely. Allergic contact dermatitis would be a consideration but is much less likely given the lack of scale and lack of pruritus. The rash does not appear like psoriasis, which is a condition with pink plaques with micaceous, thick scale. Lichen planus presents as purple polygonal papules with Wickham striae classically seen on the wrists, ankles, and mucosal sites - the rash in the picture does not represent this condition.
15- A 51-year-old man presents with a soft mass on the scalp. Which one of the following is true concerning the evaluation and management of lipomas?
A. They are usually painful and tender on examination
B. They are usually associated with a malignant condition
C. They can be watched without surgical intervention and can be familial
D. They usually require surgical removal to prevent further complications
E. They are associated with Addison disease
Correct choice: C. They can be watched without surgical intervention and can be familial
Explanation: Lipomas are benign lesions and can be familial. Lipomas are benign and do not require surgical intervention. They are not usually painful and are not associated with malignant conditions or Addison disease.
16- worker presents at your occupational dermatology clinic with the development of a white finger in response to cold, associated transient loss of sensation, and permanent finger neuropathy and pain in the affected limb. He operates chainsaws and pneumatic tools. Which of the following is the most likely diagnosis for this patient’s condition?
A. Raynaud phenomenon
B. Scleroderma
C. Mixed connective tissue disease
D. Hypothenar hammer syndrome
E. Vibration white finger
Correct choice: E. Vibration white finger
Explanation: Vibration white finger is a relatively frequent disorder among operators of chainsaws, pneumatic tools, and hand grinders who work in cold climates. It is characterized by development of a white finger or fingers in response to cold, associated transient loss of sensation, possible permanent finger neuropathy and pain in the affected limb. It is associated with exposures to vibrations between 30 and 300 Hz.
Raynaud phenomenon: In contrast to the symmetric distribution seen in Raynaud disease, in vibration white finger the blanching is asymmetric and occurs only on those digits most exposed to
vibration. Hypothenar hammer syndrome: The hypothenar hammer syndrome results from occlusion of the ulnar artery as a consequence of repeated trauma to the palms and may be misdiagnosed as Raynaud phenomenon.
17- These lesions typically erupt in what condition?
A. Morphea
B. Leprosy
C. Erythema nodosum
D. Sarcoid
E. Pregnancy
Correct choice: E. Pregnancy
Explanation: This is a pyogenic granuloma. They can erupt in pregnancy.
18- Similar lesions are present on the contralateral leg, and this patient also reports joint pain and tarry stools. A biopsy for direct immunofluorescence is most likely to show which of the following?
A. IgA deposition in the dermal papillae
B. Perivascular IgA deposition
C. Perivascular IgG deposition
D. Linear IgA deposition along the basement membrane zone
E. Linear IgG deposition along the basement membrane zone
Correct choice: B. Perivascular IgA deposition
Explanation: The associated image depicts the classic “palpable purpura” of cutaneous small-vessel vasculitis (CSVV). A specific type of CSVV now known as IgA vasculitis (formerly Henoch- Schonlein Purpura) typicallt presents with palpable purpura on the lower extremities and buttocks, along with arthritis, hematuria, colicky abdominal pain +/- GI bleeding and vomiting. A biopsy for H&E reveals leukocytoclastic vasculitis, and a biopsy for DIF shows perivascular IgA, C3, and fibrin. Treatment is mainly supportive as it is typically self-limited. Dermatitis herpetiformis - IgA deposition in the dermal papillae. Linear IgA bullous disease/chronic bullous disease of childhood - Linear IgA deposition along the basement membrane zone. Bullous pemphigoid - Linear IgG (and C3) deposition along the basement membrane zone.
19- This patient had significantly elevated serum CPK. The likely diagnosis is:
A. Dermatomyositis
B. Lupus erythematosus
C. Psoriasis
D. Atopic dermatitis
E. Lichen planus
Correct choice: A. Dermatomyositis
Explanation: The answer is dermatomyositis with theq heliotrope color and distribution of erythema or violaceous color. The skin over the metacarpal and proximal interphalangeal joints can become inflamed and erythematous forming Gottron’s papules.
20- A patient has new-onset shortness of breath and the rash shown. She denies weakness or any other systemic symptoms. Which of the following is the most likely diagnosis?
A. Limited systemic sclerosis
B. Diffuse systemic sclerosis
C. Dermatomyositis
D. Systemic lupus erythematosus
E. Sjögren’s syndrome
Correct choice: C. Dermatomyositis
Explanation: Dermatomyositis (DM) can present as clinically amyopathic disease featuring characteristic skin changes and interstitial lung disease.
This question asks the examinee to identify DM (choice 3) presenting with pathognomonic cutaneous findings without gross muscle involvement, but with shortness of breath concerning for interstitial lung disease (ILD). The photo depicts ragged cuticles (Samitz sign), violaceous papules overlying joints (Gottron’s papules), and a violaceous scaly plaque on the knee (Gottron’s sign). Anti-MDA5 (CADM-140) is an antibody associated with clinically amyopathic DM with ILD, which may be quickly progressive. While multiple rheumatologic conditions cause dyspnea by various mechanisms, their skin findings will differ from those of DM. Limited and diffuse systemic sclerosis (SS, choices 1 and 2) would show cutaneous sclerosis (confined to distal extremities and
face in limited SS; extending to proximal extremities and trunk in diffuse SS), Raynaud’s phenomenon, and nailfold capillary changes. On the hands, systemic lupus erythematosus (choice 4) favors the phalanges and spares the joints, opposite to the pattern of DM. Sjögren’s syndrome (choice 5) can manifest with xerosis, purpura, vasculitis, annular erythema, Sweet syndrome, erythema nodosum, nodular amyloidosis, and other findings distinct from those of DM.
21- A 57 year-old-female is post-op day 2 from a knee replacement. Dermatology is consulted for the dermatitis seen here. It is pruritic, and has not responded to 1% hydrocortisone. What is the most likely etiology of the physical findings?
A. Allergic contact dermatitis to a soap or other topical application
B. Irritant contact dermatitis due to excessive rubbing during the cleansing process
C. Allergic contact dermatitis to the metal within the joint replacement
D. New-onset psoriasis
E. Phototoxic drug eruption to doxycycline given pre-operatively
Correct choice: A. Allergic contact dermatitis to a soap or other topical application
Explanation: The sharp demarcation and linear almost “dripping” of the proximal aspect of the dermatitis suggest an allergic contact dermatitis particularly to a topical application. Often stronger steroids than a 1% hydrocortisone are needed. Irritant contact dermatitis is less pruritic, and less well-demarcated. Allergic dermatitis to a metal and psoriasis both will not be as well-demarcated and geometric. A phototoxic drug eruption could unlikely be on the hip and nowhere else, and is unlikely to have that “dripping” seen.
Reference: Bolognia, Jean., Jorizzo, Joseph L.Schaffer, Julie V. (Eds.) (2012) “Allergic Contact Dermatitis.” Dermatology /[Philadelphia] : Elsevier Saunders.
22- Atopic dermatitis is associated with all except:
A. Ichthyosis hystrix
B. Central facial pallor
C. Pityriasis alba
D. Nipple eczema
E. Hyperlinear palms
Correct choice: A. Ichthyosis hystrix
Explanation: Icthyosis vulgaris (not hystrix) is associated with atopic dermatitis as one of the minor criteria of Hanifin. The other listed answers are associated with atopic dermatitis.
23- What is the most likely diagnosis?
A. Keratoacanthoma
B. Lentigo maligna melanoma
C. Amelanotic melanoma
D. Merkel cell carcinoma
E. Basal cell carcinoma
Correct choice: E. Basal cell carcinoma
Explanation: Basal cell carcinoma is the most common skin cancer diagnosed and is the most likely diagnosis.
Keratoacanthoma presents as a pink to red dome shaped papule with a central keratotic core. Lentigo maligna melanoma presentas an irregular tan to brown patch. Amelanotic melanoma is possible based on the picture but is much less common than basal cell carcinoma. Merkel cell carcinoma presents as a red to purple papule.
24- What is the most likely diagnosis in this patient with a monoclonal gammopathy?
A. Sarcoidosis
B. Leprosy
C. Phymatous rosacea
D. Necrobiotic xanthogranuloma
E. Cutaneous tuberculosis
Correct choice: D. Necrobiotic xanthogranuloma
Explanation: The patient has necrobiotic xanthogranuloma (NXG) associated with a monoclonal gammopathy, which is most often an IgG kappa or lambda monoclonal gammopathy. NXG is a rare, chronic, progressive granulomatous disorder. It manifests as yellowish plaques and nodules most commonly in the mid face to periocular region.
Sarcoidosis presents more commonly as pink predominant papules with a yellow tinge on the nose and nasal columella and is not classically associated with a monoclonal gammopathy. Leprosy is an infectious cause of leonine facies but is not associated with pink granulomatous plaques on the face with an associated monoclonal gammopathy. Phymatous rosacea presents on the nose to medial cheeks with sebaceous gland hyperplasia leading to lobular and sebaceous, disfiguring appearance to the nose. Cutaneous tuberculosis is not associated with monoclonal gammopathy.
25- What is the most common area affected in multifocal fixed drug eruption?
A. Oral or genital mucosa
B. Central face
C. Upper arms
D. Thighs
E. Lower abdomen
Correct choice: A. Oral or genital mucosa
Explanation: Over half of FDE lesions occur in the oral or on the genital mucosa.
26- Which of the following substances is known to cause a delayed positive patch test reaction?
A. Gold
B. Nickel
C. Bacitracin
D. Fragrance mix
E. Rosin
Correct choice: A. Gold
Explanation: Gold is known to cause a delayed patch test reading, with some recommending a final patch test reading at 3 weeks to avoid missing any positive reactions. Patients allergic to gold often also react to nickel and cobalt. Bacitracin is a frequent contact allergen that often coexists with an allergy to neomycin. Fragrance mix contains cinnamic alcohol and aldehyde, hydroxycitronellel, isoeugenol, eugenol, oak moss absolute, alpha-amyl cinnamic aldehyde, geraniol. Rosin (colophony) is found in adhesive tape, cosmetics, glossy papers, violin bows, and chewing gum.
27- In addition to gentle washing of the face two times a day, what is the most appropriate treatment step for this patient who is prone to dry skin?
A. Start Differin 0.1 gel nightly to every other night after wash
B. Aggressive exfoliation daily
C. Start doxycycline 100 mg by mouth two times a day with food and water
D. Monotherapy with clindamycin lotion every morning after wash
E. Start isotretinoin after checking labs and a pregnancy test
Correct choice: A. Start Differin 0.1 gel nightly to every other night after wash
Explanation: The patient has comedonal acne. She should wash her face 2 times a day with a gentle cleanser. She should start using Differin 0.1 gel nightly to her face after washing. If too drying, the patient can reduce usgae of Differin to every other night. A topical retinoid treats comedones and assists with the proper keratinization of the hair follicle.
Aggressive exfoliation can damage the skin and make acne worse. Doxycycline is a systemic treatment option in cases of inflammatory acne that has not responded to combined topical treatment with clindamycin/benzoyl peroxide in the AM and a topical retinoid in the PM. Monotherapy with clindamycin is ill advised in cases of inflammatory acne as bacterial resistance becomes a problem; the patient should simultaneously used a benzoyl peroxide product. Isotretinoin is the not the first treatment in cases of comedonal acne.
28- A 32-year-old man presents with several annular and polycyclic lesions commencing in the flexures. His skin condition has a very cyclic course, as pustules resolve they are replaced by superficial scaling and then new pustules form again. He also has an associated IgA paraproteinemia. What is the most likely diagnosis?
A. Sneddon Wilkinson disease
B. Pustular psoriasis
C. Acute generalized exanthematous pustulosis
D. Superficial folliculitis
E. Benign familial pemphigus
Correct choice: A. Sneddon Wilkinson disease
Explanation: Sneddon Wilkinson is characterized by annular or polycyclic lesions, usually commencing in the flexures. Very superficial (subcorneal) sterile pustules are the hallmark of Sneddon–Wilkinson disease. There may be a gravity-induced demarcation in some vesiculopustules, with clear fluid superiorly and pus inferiorly. This disease has a cyclic course, i.e. as the pustules resolve they are replaced by superficial scaling and then new pustules form again. Some patients with Sneddon–Wilkinson disease have an associated IgA paraproteinemia. Its response to dapsone, combined with subcorneal pustules (in the absence of spongiform pustules), provide support for this condition being a disease entity distinct from pustular psoriasis.
Pustular psoriasis: In generalized pustular psoriasis, infiltration of neutrophils dominates the histologic picture and the clinical picture consists of sterile pustules on a background of bright erythema.
Acute generalized exanthematous pustulosis: This condition would present with mild spongiform pustulation, eosinophils and often a drug history with either a new medication or a change in drug history.
Superficial folliculitis: The clinical presentation and distribution would differ as superficial folliculitis is clinically characterized with appearance of pustules at follicular orifices and in infundibulum.
Benign familial pemphigus is an uncommon autosomal dominant disorder clinically characterized by flaccid blisters and erosions seen in intertriginous areas, especially the axillae and groin; moist, malodorous vegetations and fissures can develop.
29- A 49-year-old female presents with diffuse pustules on the hands and feet as seen here. It is incredibly pruritic and she is very uncomfortable. She has tried over the counter hydrocortisone with no success. She smokes a half pack of cigarettes per day and monthly uses cocaine. In addition to prescription medications, what is the most important point to counsel your patient on today?
A. She is at higher risk of cancer because of these cutaneous findings
B. The dermatosis is a variant of atopic dermatitis
C. Smoking is significantly related to these findings and cessation may be helpful
D. Cocaine is related to these findings and increased use worsening cutaneous symptoms
E. The dermatosis will resolve without intervention
Correct choice: c. Smoking is significantly related to these findings and cessation may be helpful
Explanation: Palmoplantar pustulosis, seen and descibed here, had been linked to smoking and smoking cessation can lead to improvement in some patients. The other options are not true regarding palmoplantar pustulosis.
30- Which of the following is NOT a cause of a saddle nose deformity?
A. Relapsing polychondritis
B. Hypohidrotic ectodermal dysplasia
C. Congenital syphilis
D. Trichorhinophalangeal syndrome
E. Wegener’s granulomatosis
Correct choice: D. Trichorhinophalangeal syndrome
Explanation: Trichorhinopharyngeal syndrome is the only condition from the mentioned conditions that is not associated with saddle nose deformity.It is associated with sparse hair, a bulbous (pear- shaped) nose and cone-shaped epiphyses. The other listed conditions can result in saddle nose deformity.
31- A 14-year-old male with moderate inflammatory and comedonal acne reports no improvement three months after he was prescribed a topical retinoid/benzoyl peroxide combination product. Which of the following is the most appropriate next step in management?
A. Increase topical retinoid strength
B. Add an oral antibiotic
C. Discuss initiation of isotretinoin
D. Discuss medication usage patterns
E. Add a topical antibiotic
Correct choice: D. Discuss medication usage patterns
Explanation: Considering high rates of medication nonadherence in acne, it is most appropriate to review a patient’s usage patterns and address factors limiting use before altering therapy.
Using a common clinical scenario, this question emphasizes to the examinee the need to assess medication adherence prior to changing therapy. A wide variety of patient-, therapy-, and healthcare system-associated factors influence treatment adherence. Therefore, it is most appropriate to first discuss with the patient how (or if) they are using their medication (choice 4) in order to discover any barriers, real or perceived, to optimal usage. Commonly, irritancy limits use of topical acne products, so it would be inappropriate to increase retinoid strength (choice 1) without first reviewing actual use. A short course of oral antibiotics (choice 2) or addition of a topical antibiotic (choice 5) may be appropriate measures if the patient is first determined to be using the prescribed topical product as intended. Isotretinoin (choice 3) is indicated for severe, recalcitrant nodulocystic acne and thus would not be appropriate to initiate prior to understanding how the patient is using their topical product and considering the full range of available treatment options.
32- A 33-year-old female presents with the shown painful lesions that appeared abruptly one week ago. She recently had gotten over an upper respiratory infection the previous week. She is currently febrile to 38.3C with malaise and headaches. You perform a biopsy to confirm your clinical suspicions. Which of the following medications should result in quick clearance of her skin lesions.
A. Doxycycline
B. Isotretinoin
C. Colchicine
D. Prednisone
E. Triamcinolone
Correct choice: D. Prednisone
Explanation: The clinical presentation most likely represents Sweet syndrome (acute febrile neutrophilic dermatosis). Classical Sweet syndrome is frequently associated with recent infections, IBD and pregnancy.
Diagnosis is made with 2 major and 2 minor criteria.
Major criteria: 1. Abrupt onset of painful erythematous plaques 2. Histopathological evidence of a dense neutrophilic infiltrate without evidence of leukocytoclastic vasculitis
Minor criteria: 1. Fever 2. Association with malignancy, inflammatory disease, pregnancy or recent URI, GI infection, vaccination 3. Excellent response to systemic steroids 4. Abnormal lab values (ESR, CRP, leukocytosis, >70% neutrophils)
33- A 24-year-old male presents with the findings seen here. The phenomenon seen on the patient’s arm is seen in many diagnoses in dermatology. The linearity of the dermatosis suggests which phenomenon?
A. Koebnerization
B. Allergic contact dermatitis
B. Pathergy
D. Malingering
E. Phototoxic eruption
Correct choice: A. Koebnerization
Explanation: Koebnerization is seen in several skin conditions including lichen nitidus, seen here. It is also referred to as an isomorphic response. The skin condition can grow within areas of trauma,
as seen by the linearity and nearby scratches in this photo. The other options are not seen with lichen nitidus.
34- A 16-year-old male presents with the findings seen here. The lesions have been present for 5 months and are asymptomatic. Mother is concerned that they are not going away. What is the most appropriate treatment?
A. Observation
B. Potent topical steroids
C. Oral acitretin
D. Topical tretinoin
E. Topical calcineurin inhibitor
Correct choice: A. Observation
Explanation: Lichen nitidus, seen here, is a benign often self-resolving dermatosis. Treatment is not needed, particularly if asymptomatic, and observation is appropriate. Topical steroids and calcineurin inhibitors can be used if lichen nitidus is pruritic, otherwise no treatment is needed. Oral acitretin and topical vitamin A medications are unlikely to be helpful.
35- Which of the following is not a formaldehyde-releasing preservative?
A. Bronopol
B. Methylchloroisothiazinolone
C. Quaternium-15
D. Imidazolidinyl urea
E. DMDM hydantoin
Correct choice: B. Methylchloroisothiazinolone
Explanation: Methylchloroisothiazinolone, also known as Kathon CG, is the only non- formaldehyde releasing preservative. It is found in cosmetics, skin/hair products, “acid” permanent waves, soaps, latex emulsions, and biocides. All the others are formaldehyde-releasing preservatives.
36- A 74-year-old female presents with a diffuse eruption as seen here. It affects nearly her entire body. She is most likely to have which additional systemic findings?
A. Elevated creatinine
B. Neutrophilia
C. Anemia
D. Seizures
E. Rsspiratory distress
Correct choice: B. Neutrophilia
Explanation:Acute generalized exanthematous pustulosis is often seen with fever and neutrophilia and can also have facial edema. Renal, bone marrow, neurologic and pulmonary involvement are rare.
37- What is the most appropriate action in this otherwise healthy patient based on the image in the photograph?
A. Punch biopsy
B. Tissue culture
C. Reassurance
D. Plain film of the foot
E. Magnetic resonance imaging of the foot
Correct choice: C. Reassurance
Explanation: The image depicts piezogenic papules, a benign entity associated with fatty connective tissue protrusion (most commonly) of the heel. Plain film of the foot will not reveal anything. Magnetic resonance imaging of the foot may provide additional detail particularly if the foot were under pressure - revealing areas of slight protrusion of fatty connective tissue - but imaging is not necessary to make this diagnosis. Punch biopsy and tissue culture are unnecessary and will do more harm than good.
38- Which of the following findings portends a favorable course in sarcoidosis?
A. Lupus pernio
B. Blau syndrome
C. Heerfordt syndrome
D. Löfgren syndrome
E. Mikulicz syndrome
Correct choice: D. Löfgren syndrome
Explanation: Löfgren syndrome is associated with a favorable prognosis in sarcoidosis.
This question tests the examinee’s knowledge of the types of cutaneous sarcoidosis and their associated prognoses. Of the answer choices, Löfgren syndrome (choice 4), which includes erythema nodosum, hilar adenopathy, arthritis, and fever, tends to run the most limited course, resolving spontaneously within 1-2 years. In contrast, lupus pernio (choice 1) is associated with more severe and longstanding systemic sarcoidosis requiring intensive treatment. Blau syndrome (familial juvenile systemic granulomatosis, choice 2) is a dominantly inherited disease that is phenotypically similar to sarcoidosis, manifesting in childhood with granulomatous inflammation of the skin, eyes, and joints. Heerfordt syndrome (uveoparotid fever, choice 3) comprises uveitis, parotitis, fever, and cranial nerve palsy, while Mikulicz syndrome (choice 5) describes involvement of lacrimal, salivary, and parotid glands; neither is as favorable as Löfgren syndrome.
39- What percentage of patients with this condition will develop systemic lupus erythematosus?
A. 0%
B. 5-15%
C. 40-50%
D. 70-80%
E. 100%
Correct choice: B. 5-15%
Explanation: 5-15% of patients with discoid lupus will develop systemic lupus erythematosus.
40- Which of the following concerning Degos’ Disease is true?
A. After undergoing multiple stages, it resolves without scarring
B. It affects women more than men
C. Gastrointestinal involvement portends a poor prognosis
D. Glucocorticoids are standard of care
E. Lab results indicate a low plasma fibrinogen level and decreased platelet aggregation
Correct choice: C. Gastrointestinal involvement portends a poor prognosis
Explanation: In Dego’s Disease (aka Malignant Atrophic Papulosis), GI involvement portends a poor prognosis as death from this condition is usually due to fulminant peritonitis caused by multiple perforations of the intestines.
Dego’s Disease (aka Malignant Atrophic Papulosis): It most frequently affects men and is a potentially fatal obliterative arteritis. After undergoing multiple stages, the patient is left with varicelliform scars. Later, anemic infarcts involve the intestines to produce acute abdominal symptoms of epigastric pain, fever, and hematemesis. Lab results indicate a high plasma fibrinogen level and increased platelet aggregation. Administration of corticosteroids has not been beneficial.
41- A 45-year-old man presents for evaluation of 3 months of intensely pruritic violaceous papules over the wrists, ankles, and genitals. Which of the following elements of the history is most important in establishing the diagnosis?
A. Hepatitis B immunization status
B. Sexual history
C. Medication history
D. Family history
E. Age-appropriate malignancy screening
Correct choice: C. Medication history
Explanation: For a new-onset lichenoid eruption, it is crucial to review the medication history and consider the possibility of lichenoid drug eruption, which can be clinically and histopathologically identical to idiopathic lichen planus (LP).
This question tests the examinee’s knowledge of pertinent history to obtain for a new-onset lichenoid eruption. In this scenario, it is essential to review both prescription and nonprescription medication use (choice 3) and consider the possibility of lichenoid drug eruption, which can be clinically and histologically indistinguishable from idiopathic LP. Rather than hepatitis B (choice
1), hepatitis C status is important to determine, more so in specific populations and with oral LP. LP is not related to sexual activity (choice 2). Familial LP (choice 4) is relatively uncommon and less useful to review than medications. As LP is not a paraneoplastic phenomenon, malignancy screening (choice 5) would not be relevant in this situation.
42- A 60-year-old male patient on dialysis develops a fixed livedo reticularis which is firm to the touch. The areas become increasingly violaceous and purpuric, eventually bullous and necrotic. The patient also similar lesions on his abdomen, buttocks and thighs. He complains of severe pain, and requires analgesia for control. What is the most likely diagnosis?
A. Livedo racemosa
B. Calciphylaxis
C. Cryoglobulinemia
D. Disseminated intravascular coagulation
E. Hematoma
Correct choice: B. Calciphylaxis
Explanation: Calciphylaxis begins as fixed livedo reticularis (livedo racemosa), which is frequently firm or hard to the touch. Areas within the livedo become increasingly violaceous and eventually purpuric, bullous, and necrotic. Affected tissue has reduced oxygenation. Lesions affect the legs below the knees in 90% of patients. More proximal lesions and those of the fatty areas of the thighs, buttocks, and abdomen occur in about two-thirds. Severe pain is a cardinal feature of calciphylaxis, often requiring narcotic analgesia for control.
1 - Livedo racemosa refers to a form of LR that has a larger, branching, and more irregular pattern and is often more widespread, affecting both the extremities and the trunk
3 - Cryoglobulins are cold-precipitable immunoglobulins that can be divided into three subtypes, all of which have cutaneous manifestations including palpable purpura, myalgias and arthralgia, peripheral neuropathy and glomerulonephritis.
4 - Disseminated intravascular coagulation is a condition in which small blood clots develop throughout the bloodstream, blocking small blood vessels. The increased clotting depletes the platelets and clotting factors needed to control bleeding, causing excessive bleeding.
5 - Hematoma is a solid swelling of clotted blood in the tissues
43- Which of the following is not true of this condition?
A. Associated with twin gestation
B. Associated with primigravidas
C. Associated with premature labor
D. Associated with maternal obesity
E. Associated with third trimester of pregnancy
Correct choice: C . Associated with premature labor
Explanation: The figure illustrates polymorphic eruption of pregnancy (PUPPP) with erythematous papules within striae. There is no associated maternal or fetal risk. All of the remaining choices are true.
44- What test should be performed for our patient with the hair loss pictured?
A. Serum zinc level
B. RPR
C. ESR and CRP
D. Serum copper level
E. Free and total testosterone levels
Correct choice: B. RPR
Explanation: Patient has moth-eaten alopecia of secondary syphilis. The correct answer is RPR to screen for syphilis. Copper and zinc levels are not necessary. The hair loss is not consistent with androgenetic alopecia, but even if it were you do not need free testosterone and total testosterone levels to make that diagnosis. ESR and CRP are non-specific tests to evaluate for inflammation in the body and are not useful in this setting.
45- Which of the following is the best treatment for this condition?
A. Brimonidine gel
B. Oxymetazoline cream
C. Hydrocortisone ointment
D. Ketoconazole cream
E. Doxycycline
Correct choice: E. Doxycycline
Explanation: This is peri-oral dermatitis, which is best treated with doxycycline of these choices. Several therapies, including oral antibiotics (e.g., tetracyclines), topical antibiotics (e.g., erythromycin, clindamycin), topical metronidazole, topical azelaic acid, topical sulfacetamide-
sulfur, and topical calcineurin inhibitors (e.g., tacrolimus, pimecrolimus), have been used with some success to treat peri-oral dermatitis. Brimonidine gel and oxymetazoline cream are used to treat the persistent erythema of rosacea. While many believe that peri-oral dermatitis may represent a form of rosacea, brimonidine gel and oxymetazoline cream will not treat the papules and pustules seen in the image here. Hydrocortisone ointment (and other topical steroids) typically worsen peri-oral dermatitis and thus should be avoided. Ketoconazole cream is a topical anti-fungal that does not treat peri-oral dermatitis.
47- Which of the following conditions is most likely associated with these cutaneous findings?
A. Hypertension
B. Ulcerative colitis
C. Congestive heart failure
D. Diabetes
E. Cataracts
Correct choice: D. Diabetes
Explanation: Granuloma annulare is a benign, non-infectious, granulomatous condition of the skin that is usually asymptomatic and self-limited. It typically presents with annular erythematous plaques that are most commonly found on the dorsal surfaces of the hands and feet. Generalized and perforating variants are most closely associated with diabetes.
48- A 66-year-old male who lives alone presents for a dark rash on the back that his PCP was concerned about. What is the best treatment for the findings seen on this patient’s back?
A. Metronidazole gel
B. Scrubbing with an alcohol swab
C. Methotrexate
D. Oral prednisone
E. Clindamycin solution
Correct choice: B. Scrubbing with an alcohol swab
Explanation: This is an example of terra firma-forme or Duncan’s dirty dermatosis. It presents as dirt-like plaques and is both diagnosed and treated with isopropol alcohol. The other options have not been shown to help treat terra firma-forme.
49- An HIV positive male presents with psoriasis over 8% of his body (not face or scalp), he has not tried any prescription medications. He is not using moisturizers. What is the correct initial approach?
A. Start cyclosporine and monitor renal function and blood pressure
B. Test the patient for tuberculosis and if negative start Humira (adalimumab) subcu injection 80 mg initial dose on day 1 followed by 40 mg subcu injection on day 8 and 40 mg subcu every two weeks thereafter
C. Begin acitretin 25 mg PO daily after checking CBC, CMP, lipid panel
D. Start mometasone 0.1 ointment BID to affected areas on body, advise face to feet moisturizer at least BID
E. Begin Otezla (apremilast) starter pack by mouth as directed with goal to titrate up to 30 mg two times a day
Correct choice: D. Start mometasone 0.1 ointment BID to affected areas on body, advise face to feet moisturizer at least BID
Explanation: The correct answer is to begin topical steroid two times a day to the rash and use emollient at least two times a day to the body in patient with 8% BSA psoriasis who has not tried any prescription treatments. While the other options are treatments that could be used in for psoriasis in a patient with HIV and psoriasis, it is most appropriate to start with topical steroid and emollient for treatment. Cyclosporine is not a chronic medication for psoriasis and is not a medication to use for 8% BSA psoriasis.
50- A 32 year old male patient presents with the sudden eruption of multiple itchy bumps on his feet after playing with his children in the grass. Hydrocortisone is improving the itchiness somewhat. What is the most likely diagnosis?
A. Dermatitis herpetiformis
B. Chigger bites
C. Neurotic excoriations
D. Allergic contact dermatitis
E. Porphyria cutanea tarda
Correct choice: B. Chigger bites
Explanation: The patient has encountered chiggers, mite larvae who feed on human keratinocytes. Their feeding process induces the development of the characteristic chigger bite, which is very pruritic. Topical steroids can reduce the itch sensation. Dermatitis herpetiformis commonly presents on extensor surfaces as pruritc papules and vesicles with excoriation. The onset would not be as sudden and limited to the feet. The lesions are not excoriations (a secondary skin lesion) and thus neurotic excoriations is incorrect. Allergic contact dermatitis would present as ill-defined eczematous pink patches with scale, not as discrete papules. Porphyria cutanea tarda would show
evidence of vesicles, scars, and milia - none of which are present in this photograph. In addition, the condition is chronic.
51- Which one of the following can develop into squamous cell skin cancer over time?
A. Actinic keratosis
B. Onychomycosis
C. Seborrheic keratosis
D. Psoriasis
E. Impetigo
Correct choice: A. Actinic keratosis
Explanation: Actinic keratosis can develop into squamous cell skin cancer, thus it is a pre-cancerous lesion. Seborrheic keratosis is a benign thickening of the skin not associated with a pre-malignant states. Onychomycosis is a fungal infection of the nail not associated with a pre-malignant state. Psoriasis is an inflammatory skin condition not associated with a pre-malignant state. Impetigo is a superficial infection of the skin by bacteria and is not associated with a pre-malignant state.
52- Autoantibodies to ECM-1 are found in which of the following conditions?
A. Lipoid proteinosis
B. Lichen sclerosus
C. Focal dermal hypoplasia
D. Incontinentia pigmenti
E. Marfan syndrome
Correct choice: B. Lichen sclerosus
Explanation: Autoantibodies to extracellular matrix protein-1 (ECM-1) are found in lichen sclerosus (LS). This question tests the examinee’s knowledge of the pathogenesis of LS. In LS (choice 2), up to 80% of patients have circulating autoantibodies to ECM-1, a glycoprotein that contributes to integrity of the basement membrane zone and assembly of collagen fibrils. An important distinction needs to be made between LS and lipoid proteinosis (choice 1), in which there is an ECM1 gene
mutation. The remaining answer choices are caused by distinct gene mutations: PORCN in focal dermal hypoplasia (Goltz syndrome, choice 3); IKBKG/NEMO in incontinentia pigmenti (choice 4); and fibrillin 1 in Marfan syndrome (choice 5).
53- Patients with this disorder may develop exaggerated reactions to insect bites:
A. Chronic lymphocytic leukemia
B. Bullous pemphigoid
C. Lupus erythematosus
D. Dermatomyositis
E. Incontinentia pigmenti
Correct choice: A. Chronic lymphocytic leukemia
Explanation: Patients with chronic lymphocytic leukemia may develop exaggerated reactions to insect bites, including bullous reactions. Patients with the other listed conditions to do exhibit exaggerated reactions to insect bites.
54- On patch testing, which of the following is most likely to produce a relevant positive reaction?
A. Budesonide
B. Gold sodium thiosulfate
C. Thimerosal
D. Epoxy resin
E. Paraben mix
Correct choice: B. Gold sodium thiosulfate
Explanation: Several studies have found gold to be the most common relevant culprit in eyelid allergic contact dermatitis (ACD).
This question assesses the examinee’s knowledge of ACD involving the eyelids. Common eyelid contact allergens include metals (most prominently gold [choice 2], nickel, and cobalt), fragrances, preservatives, topical antibiotics, and surfactants. None of the other options are among the most common causes of eyelid ACD: budesonide (choice 1) is the screening agent for group 1 topical corticosteroids; thimerosal (choice 3) is a preservative that uncommonly causes ACD (although it is found in some ophthalmic drops); epoxy resin (choice 4) is found in adhesives; and paraben mix (choice 5) screens for parabens, common preservatives in cosmetics and personal care products that rarely cause ACD.
55- A 42-year-old man presents with milky white macules and patches in his periorbital area surrounded by normal skin. The lesions have discrete margins and are irregular in shape. On Wood’s lamp examination, the contrast between the affected areas and the surrounding skin is striking. Which o the following statements is not true in this condition?
A. Vitiligo vulgaris is the most common clinical type of vitiligo observed in children
B. Segmental vitiligo is significantly increased in adults compared to children
C. In lesions of vitiligo, melanocytes are typically absent or present in small numbers
D. The epidermal melanocyte density can be assessed with Melan-A, MITF, and HMB-45
E. The epidermal melanocyte density can be assessed via incubation of biopsy specimens with dihydroxyphenylalanine
Correct choice: B. Segmental vitiligo is significantly increased in adults compared to children
Explanation: Segmental vitiligo is significantly increased in children compared to adults. Although vitiligo vulgaris is the most common clinical type observed in children, the frequency of segmental vitiligo (~15–30%) is significantly increased compared to that in adults (<5–10%). The incidence of associated endocrinopathies is less than in the adult vitiligo population. A family history of vitiligo is associated with an earlier age of onset.
- Vitiligo vulgaris is the most common clinical type of vitiligo observed in children
- In lesions of vitiligo, melanocytes are typically absent or present in small numbers. The epidermal melanocyte density can be assessed with melanocyte-specific immunohistochemical stains, such as Melan-A (MART-1), MITF, and HMB45, or via incubation of biopsy specimens with dihydroxyphenylalanine (DOPA; detects tyrosinase activity). Although ultrastructural studies may be performed for research purposes, very few hypomelanotic disorders have specific ultrastructural findings.
56- Morsicatio buccarum results from which of the following?
A. Metal dental amalgams
B. Nicotine exposure
C. Melkersson-Rosenthal syndrome
D. Keratin 4 and 13 mutations
E. Chronic irritation from biting
Correct choice: E. Chronic irritation from biting
Explanation: Morsicatio buccarum is the term describing a shaggy white plaque on the buccal mucosa occurring secondary to chronic irritation from biting.
57- You are evaluating a 25-year-old woman who presents with a rash on her face. She states she might be having a reaction to a food she ate, but she is not sure. The rash is erythematous and patchy on both cheeks, and there is no sparing of the nasolabial folds. Which one of the following would you order at this point?
A. ANA, C3, and C4
B. Oral prednisone
C. Oral doxycycline
D. Oral acyclovir
E. Rheumatoid factor, ESR, and CRP
Correct choice: C. Oral doxycycline
Explanation: The patient has rosacea, a chronic inflammatory rash classically of the mid face. Rosacea does not spare the nasolabial folds, while cutaneous lupus does. Lupus spares the nasolabial fold. In the case of lupus, you would consider ordering ANA, C3, and C4 and you may start prednisone if the patient is having disease severe enough to warrant treatment with systemic steroid. Ordering rheumatoid factor, ESR, and CRP will not help the patient and oral acyclovir is not indicated as the patient does not have a viral disease such as herpes simplex or herpes zoster.
58- A 40-year-old man presents with rough papules on the dorsal aspect of the proximal fingers which are also seen on the trunk and extremities. In some areas, the papules coalesced to form large orange-red plaques. His palms and soles have an orange-red appearance, and there is erythema and diffuse fine scale of the scalp. What is the most likely diagnosis for this patient’s condition?
A. Seborrheic dermatitis
B. Cutaneous T-cell lymphoma
C. Pityriasis rubra pilaris
D. Kawasaki disease
E. Psoriasis
Correct choice: C. Pityriasis rubra pilaris
Explanation: Pityriasis rubra pilaris characteristically presents with follicular papules with an erythematous base, including on the proximal dorsal fingers. There is a coalescence of orange–red plaques, but with obvious islands of sparing. An orange–red waxy keratoderma of the palms and soles is often seen.
1 - Seborrheic dermatitis: Early PRP of the scalp can mimic seborrheic dermatitis. As additional clinical features of PRP develop, these two entities can be distinguished. PRP has additional distinguishing features such as follicular papues, orange-red plaques, islands of sparing, and waxy keratoderma. Scalp dermatitis is much more responsive to conventional therapy. The relative recalcitrant nature of scalp PRP may serve as a clue to the diagnosis.
2 - Cutaneous T-cell lymphoma: A PRP-like eruption can be seen in patients with dermatomyositis (Wong type) and cutaneous T-cell lymphoma, as can erythema and scaling of the scalp however the other clinical findings are characteristic of pityriasis rubra pilaris.
4 - Kawasaki disease: Children with acute-onset PRP may be misdiagnosed as having Kawasaki disease, however the older age of this patient and the other associated clinical features present do not support this diagnosis.
5 - Psoriasis: The major entity in the differential diagnosis of pityriasis ruby pillars is psoriasis. The distinctive orange–red palmoplantar keratoderma plus the keratotic follicular papules with a nutmeg grater appearance, the classic islands of sparing of the trunk, a finer scale, and no family history of psoriasis help differentiate PRP from psoriasis. The presence of oil-drop changes, small pits, and marginal onycholysis of the nails favor psoriasis.
59- Which of the following is associated with this condition?
A. Regular nail pitting
B. White dermatographism
C. Decreased risk for cutaneous infections
D. Scarring hair loss
E. Inflammatory arthritis
Correct choice: B. White dermatographism
Explanation: The photo is of Dennie-Morgan lines in a chronic atopic dermatitis patient. Regular nail pitting is seen in psoriasis; atopic dermatitis has increased cutaneous infections; scarring hair loss and inflammatory arthritis are not associated with atopic dermatitis.
60- Which of the following is TRUE regarding this condition?
A. 50% of patients will eventually develop systemic involvement
B. C3 deficiency increases susceptibility to this condition
C. Biopsy typically shows a thinned basement membrane
D. Squamous cell carcinoma is a potential sequela
E. UV exposure is helpful in treatment
Correct choice: D. Squamous cell carcinoma is a potential sequela
Explanation: This is an image of discoid lupus erythematosus (DLE). Squamous cell carcinoma may develop from lesions of DLE. SCCs arising from lesions of DLE are rare and aggressive tumors with greater likelihood of metastases. Cases have been reported among patients with different clinical characteristics. Classically, 5-15% of patients with DLE will eventually develop SLE. C2 (not C3) deficiency increases susceptibility to autoimmune conditions, particularly lupus (DLE, SCLE, SLE). Biopsy characteristically shows a thinned epidermis with plugged follicles, vacuolar degeneration of the basal layer, a thickened basement membrane, melanin incontinence, perivascular/periadnexal lymphocytes, and increased mucin deposition between collagen bundles. UV exposure exacerbates DLE, so UV protection is a critical aspect of treatment.
61- A 30 year old pregnant woman presents with a red lesion near her nail that has rapidly appeared. She is in her third trimester, and has had a normal pregnancy course to date. Which of the following is the most likely diagnosis?
A. Melanoma
B. Glomus tumor
C. Bacillary angiomatosis
D. Pyogenic granuloma
E. Inflamed wart
Correct choice: D. Pyogenic granuloma
Explanation: A pyogenic granuloma is one of several vascular lesions that can present on the skin during pregnancy, and they often develop rapidly in a peri-ungual location. At present, the pathogenesis of pyogenic granuloma is unknown. In addition to pregnancy, reported triggers include antecedent trauma and certain drugs, including acitretin, HIV protease inhibitors, EGFR inhibitors, docetaxel, capecitabine and rituximab. In the case of pyogenic granulomas arising during pregnancy, they often regress after delivery and treatment is not usually required unless they cause excessive bleeding. Other vascular lesions that can develop during pregnancy include palmar erythema, spider angiomas, varicose veins, cavernous hemangiomas, glomus tumors, and hemorrhoids. The development of vascular lesions during pregnancy is thought to be due to a combination of increased hormones and increased intravascular pressure. Of the answer choices listed, a pyogenic granuloma is the most likely diagnosis based on the clinical description.
62- Which of the following drug(s) are commonly implicated in drug-induced vasculitis?
A. Propylthiouracil
B. Adalimumab
C. Propranolol
D. Procainamide
E. Dapsone
Correct choice: A. Propylthiouracil
Explanation: PTU is well-reported to cause p-ANCA positive vasculitis.
63- What is the best medication choice for the condition pictured that involves 8% body surface area (BSA) only on the legs, no prior treatment tried.
A. Total body nbUVB treatment 3 times per week (covering genitals and eyes with shields)
B. Tremfya by injection (starter dose then standard maintenance)
C. Remicade by infusion (every 8 weeks)
D. Halobetasol 0.05% ointment 2 x a day to areas with rash
E. Otezla by mouth (uptitration via starter pack then standard dose of 30 mg 2 x a day by mouth)
Correct choice: D. Halobetasol 0.05% ointment 2 x a day to areas with rash
Explanation: The patient has psoriasis with BSA 8% with no treatment trial previously. The most reasonable next step is to begin a potent topical steroid on the areas with psoriasis on the legs 2 x a day. You can consider occlusion of the steroid at night if patient not responsive to topical steroid or if the steroid tends to rub off at night onto the bed.
The patient does not have a BSA% greater than 10 and thus choices such as Tremfya and Remicade are an over-treatment in this patient who has not trialed any conservative measures yet. If patient had psoriatic arthritis, you should then entertain biologic treatment but not for psoriasis vulgaris with BSA 8% on the legs. nbUVB to the whole body 3 x a week is over-treatment for a patient with such limited skin disease of 8% on the legs. You could consider Excimer treatment (handheld nbUVB targeted treatment for smaller disease areas on the skin) if you had the device in your office but that was not one of the choices. Much like Tremfya and Remicade, Otezla would represent an over-treatment for the patient in the vignette but you could consider using the medication if the patient also had psoriatic arthritis.
64- A 45 year old man presents to clinic with a history of Crohn’s disease, joint stiffness in the morning, and these skin findings. Which of the following is the best treatment choice for this patient?
A. Secukinumab
B. Ixekizumab
C. Guselkumab
D. Ustekinumab
E. Apremilast
Correct choice: D. Ustekinumab
Explanation: This patient has psoriasis, psoriatic arthritis, and Crohn’s disease. Ustekinumab binds the p40 subunit to inhibit IL-12 and IL-23. Ustekinumab is approved for the treatment of plaque psoriasis, psoriatic arthritis, and Crohn’s disease. It is the best treatment choice. A higher dose of ustekinumab is needed to treat Crohn’s disease than to treat psoriasis.
Secukinumab is a monoclonal antibody that inhibits IL-17A. It is FDA-approved to treat psoriasis and psoriatic arthritis, but like other IL-17 inhibitors (e.g. Ixekizumab) should not be used in Crohn’s disease. Guselkumab inhibits IL-23 by selectively binding the p19 subunit of IL-23. It was FDA-approved in July 2017 to treat plaque psoriasis only. Apremilast is approved for the treatment of psoriasis and psoriatic arthritis, but not Crohn’s disease (thereby making Ustekinumab the better choice).
65- What is the most likely diagnosis?
A. Psoriasis
B. Pityriasis rosea
C. Cutaneous T-cell lymphoma
D. Contact dermatitis
E. Lichen planus
Correct choice: D. Contact dermatitis
Explanation: This periumbilical eczematous eruption is classic for allergic contact dermatitis secondary to nickel exposure. The metal snaps on the pants are the cause of the nickel exposure in this case. The remaining answer choices are less likely, especially given that the image shows a well-demarcated eczematous periumbilical eruption and metal snaps suggesting a contact nickel allergy.
66- A 27-year-old woman presents with an eczematous rash on the earlobes after wearing new earrings. She mentions that she gets the same kind of rash when she wears a metal belt. Which of the of the following metals should she most likely avoid?
A. Stainless steel
B. White gold
C. Gold (24 ct)
D. Sterling silver
E. Platinum
Correct choice: B. White gold
Explanation: The case presented is consistent with a contact dermatitis, likely allergic contact dermatitis to a metal. The most common allergen is nickel. White gold may contain nickel, therefore this type of metal should be avoided. Patient’s allergic to nickel, typically can tolerate gold 18 ct or higher, stainless steel, sterling silver, and pure platinum.
67- A 37-year-old female presents with this eruptions, most prominent on the arms and legs. It is slightly pruritic, but the appearance is most concerning to her. What is the most likely diagnosis?
A. Cutaneous Crohns disease
B. Necrobiosis lipoidica
C. Granuloma annulare
D. Nummular eczema
E. Pityriasis rosea
Correct choice: C. Granuloma annulare
Explanation: The dermatosis seen here is annular, with erythematous and raised borders and is slightly pruritic. Of the choices, granuloma annulare is the most likely diagnosis. The alternative options do not represent the clinical granulomatous disease of granuloma annulare.
68- Which of the following is the most appropriate initial treatment for the condition pictured?
A. Prednisone
B. Mycophenolate mofetil
C. Infliximab
D. Cyclosporine
E. Doxycycline
Correct choice: A. Prednisone
Explanation:Prednisone is the most appropriate initial treatment for neutrophilic dermatosis of the dorsal hands.
The examinee is presented with the image of an eroded violaceous plaque on the dorsal hand that is highly consistent with neutrophilic dermatosis of the dorsal hands, an entity combining clinical and pathologic features of Sweet’s syndrome and pyoderma gangrenosum. Prednisone (choice 1) is the most appropriate first-line treatment. While the remaining options (choices 2-5) have been used as
second-line and beyond treatment of Sweet’s syndrome and/or pyoderma gangrenosum, none of them represents the most appropriate first-line therapy.
69- Hydrogen peroxide is often chosen for treatment of which of the following?
A. Cleaning a chronic wound on the lower extremity
B. Treatment of a chronic venous ulcer
C. Post-operative follow-up of a full thickness skin graft with a bolster
D. Cutaneous fungal infections
E. Acne vulgaris
Correct choice: C. Post-operative follow-up of a full thickness skin graft with a bolster
Explanation: Hydrogen peroxide is cytotoxic and therefore not a first choice treatment in cleaning chronic wounds and ulcers. Hydrogen peroxide is use to remove crusts on healing wounds such as bolster on a skin graft.
70- Which of the following does NOT predict mortality with the pictured eruption?
A. Age
B. Serum bicarbonate
C. Serum glucose
D. Serum urea level
E. Serum protein level
Correct choice: E. Serum protein level
Explanation: Mortality from Stevens-Johnson Syndrome/toxic epidermal necrolysis is predicted by the SCORTEN scoring system. Prognostic factors include age, heart rate, presence of cancer/ malignancy, BSA involved > 10% on day 1, serum urea level, serum glucose level, and serum bicarbonate level.
71- A 32-year-old man originally from southern China presents to your clinic with rash (pictured) and complaints of muscle weakness. Which of the following is the most likely underlying malignancy responsible for his presentation?
A. Lung adenocarcinoma
B. Colorectal adenocarcinoma
C. Gastric adenocarcinoma
D. Nasopharyngeal carcinoma
E. Testicular germ cell tumor
Correct choice: D. Nasopharyngeal carcinoma
Explanation: Muscle weakness coupled with erythematous papules overlying the dorsal hand joints (Gottron’s papules) is highly suggestive of dermatomyositis, which can be a paraneoplastic phenomenon. Nasopharyngeal carcinoma, one of the most common cancers in southern China and Southeast Asia, was the most common associated cancer in Asian studies of dermatomyositis. The remaining malignancies may produce dermatomyositis as a paraneoplastic phenomenon, but they are less likely in a man originally from southern China.
72- A 65-year-old female patient presents with poorly demarcated, symmetric, very painful patches of erythema and retiform purpura, favoring the thighs and buttocks. Bullae and a dusky gray discoloration developed, as well as the appearance of ulcerations with black, leathery eschars. A skin biopsy of the affected area is suggestive of calciphylaxis. Shortly after, she dies from secondary infection and sepsis. Which of the following is NOT a known risk factor for the development of this condition?
A. Diabetes mellitus
B. Obesity
C. End-stage renal disease
D. Hemodialysis
E. Male gender
Correct choice: E . Male gender
Explanation: Known risk factors for calciphylaxis include end-stage renal disease, hemodialysis, obesity, diabetes mellitus, and female gender as well as warfarin and liver disease. Hyperparathyroidism, calcium supplementation, and vitamin D intake are additional risk factors, with the latter possibly related to the differentiation of vascular smooth muscle cells into osteoblast- like cells. Known risk factors for calciphylaxis include end-stage renal disease, hemodialysis, obesity, diabetes mellitus, and female gender as well as warfarin and liver disease.
73- Which type of calcification best characterizes the condition pictured?
A. Metastatic
B. Dystrophic
C. Iatrogenic
D. Idiopathic
E. Mixed
Correct choice: D. Idiopathic
Explanation: Scrotal calcinosis, or “idiopathic calcified nodules of the scrotum,” cannot be attributed to any identifiable local or systemic causes.
This question assesses the examinee’s ability to recognize scrotal calcinosis and discriminate between the five main types of cutaneous calcification (calcinosis cutis). Scrotal calcinosis is a common form of idiopathic calcification (choice 4), meaning that it is not related to any recognizable local or systemic etiologies. In contrast, metastatic calcification (choice 1) affects normal tissue during states of calcium/phosphate dysregulation, most commonly renal disease. Dystrophic calcification (choice 2) occurs when there is local tissue damage in the setting of normal calcium metabolism, taking place in a broad range of conditions including autoimmune
connective tissue diseases (CREST syndrome, childhood dermatomyositis), lobular panniculitides (especially pancreatic), traumas (neonatal heel sticks), genodermatoses (pseudoxanthoma elasticum), infections (parasitic primarily), and neoplasms (pilomatricomas). Iatrogenic calcification (choice 3) is induced by diagnostic or therapeutic interventions, most often by extravasation of calcium-containing intravenous solutions. Mixed calcification (choice 5) combines the features of metastatic and dystrophic calcification; calciphylaxis is a prominent example.
74- A 28-year-old patient presents with a rare, pruritic vesiculobullous eruption that developed in the immediate postpartum period after a recent childbirth. A direct immunofluorescence shows
linear C3 deposition along the basement membrane zone. Her infant was born premature and small for gestational age. Which of the following is the most likely diagnosis?
A. Prurigo of pregnancy
B. Herpes gestationis
C. Polymorphic eruption of pregnancy
D. Atopic eruption of pregnancy
E. Intrahepatic cholestasis of pregnancy
Correct choice: B. Herpes gestationis
Explanation: Herpes gestationis is also known as gestational pemphigoid or pemphigoid gestationis. It is a rare, pruritic vesiculobullous eruption that develops during late pregnancy or the immediate postpartum period. On direct immunofluorescence, there is linear C3 deposition along the basement membrane zone in 100% of patients (30% of patients also have IgG deposition). IgG1 autoantibodies are directed against a transmembrane hemidesmosomal protein (BP180, BPAG2, collagen XVII). There is an increased risk of prematurity and small-for-gestational age neonates – the risk correlates with the severity of disease. It commonly recurs in subsequent pregnancies.
1 – Prurigo of pregnancy is a benign nonspecific pruritic papular rash that arises during pregnancy. It has also been described with other names such as prurigo gestationis, early onset prurigo of pregnancy, and pruritic folliculitis of pregnancy. It typically manifests as a papular dermatitis (prurigo) and can resemble nodular prurigo. Direct immunofluorescence is negative and histology is nonspecific. It is not associated with any maternal or fetal risks, and typically resolves after delivery of the baby. 3 – Polymorphic eruption of pregnancy manifests as urticarial papules and plaques that usually appear within striae distensae during the latter portion of the third trimester or immediately postpartum. There is development of polymorphous features (vesicles, erythema, target, and eczematous lesions) with disease progression. It is most frequent in primiparous women. Direct immunofluorescence is negative, routine labs are normal, and histologic features are nonspecific. There are no maternal or fetal risks. 4 – Atopic eruption of pregnancy is characterized by eczematous or papular skin lesions in a patient with atopic diathesis in whom other specific dermatoses have been excluded. It generally appears earlier than other pregnancy-related dermatoses (75% before the third trimester). Histology is nonspecific, direct immunofluorescence is negative. Up to 70% of patients have elevated serum IgE levels. There are no maternal or fetal risks. 5 – Intrahepatic cholestasis of pregnancy is characterized by pruritus without primary skin lesions with an onset during the third trimester. Secondary changes correlate with disease duration and vary from subtle excoriations to severe prurigo nodularis. Histology is nonspecific and direct immunofluorescence is negative. Elevated total serum bile acid levels are diagnostic. There is an increased risk of prematurity, intrapartum fetal distress and stillbirths.
75- A child develops an allergic reaction at the site of an influenza vaccine. To which of the following substances may she be allergic to?
A. Lanolin
B. Thimerosol
C. Ethylenediamine dichloride
D. Triclosan
E. Gluteraldehyde
Correct choice: B. Thimerosol
Explanation: Thimerosol is a preservatives in vaccines such as the influenza, tetanus, and diphtheria vaccines. It is also found in antitoxins and immunoglobulins. Thimerosol is a mercury-containing organic compound. Lanolin is from the sebum of sheep. Ethylenediamine dichloride is a stabilizer in topical creams, medicines, dyes, insecticides, and fungicides and was previously found in nystatin cream. Triclosan is an antibacterial agent found in soap, shampoo and mouthwash. Gluteraldehyde is a cold sterilizing solution used for medical and dental equipment.
76- You have been caring for an adolescent male with acne conglobata. He presents to your clinic with osteoarticular lesions that gradually improve with anti-inflammatory medication. You suspect a diagnosis of SAPHO syndrome. Which of the following is NOT one of the osteoarticular manifestations of SAPHO syndrome?
A. Inflammatory synovitis
B. Arthrosteitis
C. Hyperostosis and osteitis of the anterior chest wall and spine
D. Septic osteomyelitis
E. Chronic recurrent multifocal osteomyelitis
Correct choice: D. Septic osteomyelitis
Explanation: SAPHO syndrome consists of a spectrum of aseptic neutrophilic dermatoses in association with aseptic osteoarticular lesions that have distinctive histologic and radiographic features. Osteoarticular manifestations include: inflammatory synovitis; arthrosteitis (inflammation of the osseous structures of a joint); hyperostosis or osteitis, most often anterior chest wall (sternum, clavicles, ribs) and axial skeleton (spine, pelvis). A subtype of chronic recurrent multifocal
osteomyelitis (CRMO) manifests as frequent involvement of metaphyses of tubular bones and is seen primarily in children.
77- A 65-year-old woman presents with yellow-brown patches with an irregular outline and pinpoint petechiae within patches in successive crops. The lesions are mostly present in the lower legs and began after a long hike. Which of the following pigmented purpuric eruptions is the most likely diagnosis in this scenario?
A. Schamberg disease
B. Lichen aureus
C. Purpura annularis telangiectoides of Majocchi
D. Pigmented purpuric lichenoid dermatitis of Gougerot and Blum
E. Eczematid-like purpura of Doucas and Kapetanakis
Correct choice: A. Schamberg disease
Explanation: Schamberg disease is the most common form of pigmented purpura. It can occur in children and has a peak frequency in middle-aged to older men. It is characterized by yellow-brown patches with an oval to irregular outline. There are pinpoint peteachiae within patches - likened to cayenne pepper. It occurs in successive crops and favors the lower legs.
2 - Lichen aureus is a rare variant of pigmented purpura which is characterized by a solitary patch or purpuric macules, papules and patches primarily on the lower extremities.
3 - Purpura annularis telangiectoides of Majocchi is uncommon and occurs in adolescents and young adults, especially women. It is characterized by 1-3 cm annular plaques that may slowly expand. There are punctate telangiectasias and petechiae within the border. The location is the trunk and proximal lower extremities.
4 - Pigmented purpuric lichenoid dermatitis of Gougerot and Blum is a rare variant of pigmented purpura that occurs in middle-aged to older men. It is an admixture of two ypes of lesions: Schamberg-like, purpuric red-brown lichenoid papules. It is chronic and occasionally pruritic.
5 - Eczematid-like purpura of Doucas and Kapetanakis is a rare variant of pigmented purpura characterized by scaly petechial or purpuric macules, papules and patches. It is pruritic and mostly on the lower extremities.
79- Which of the following is true regarding Nekam’s Disease?
A. Is generally responsive to topical and intralesional steroids
B. Characteristically lacks scale
C. Rarely involves the buttocks
D. Presents with a reticulate pattern on the dorsal hands and feet
E. Presents with hypopigmented, atrophic lesions on the extremities
Correct choice: D. Presents with a reticulate pattern on the dorsal hands and feet
Explanation: Nekam’s Disease (keratosis lichenoides chronica) presents with violaceous papules and nodules, hyperpigmented and hyperkeratotic, covered with gray scales. There is often a linear and reticulate pattern on the dorsal hands and feet, extremities and buttocks. This condition is generally very refractory to treatment. The other answer choices listed are incorrect.
80- Pediatric patients that present with this deformity should be monitored for which of the following sequela?
A. Seizures
B. Sclerodactyly
C. Raynaud’s phenomenon
D. Cutaneous calcification
E. Photosensitivity
Correct choice: A. Seizures
Explanation: En coup de sabre means “stroke or blow of the sword” and is a term for linear morphea of the scalp/forehead. Rarely this can affect underyling structures including muscle, bone, meninges and brain and can create a focus for seizures.
Sclerodactyly, Raynaud’s phenomenon and cutaneous calicification are seen in systemic scleroderma or CREST syndrome, not linear morphea which is typically limited to the skin. Photosensitivity is not a feature of linear morphea.
81- Biopsy of this lesion reveals massive papillary dermal edema, a diffuse neutrophilic infiltrate, and a lack of vasculitis. Stains for infectious organisms are negative. What is the first line treatment?
A. Dapsone
B. Colchicine
C. Thalidomide
D. Potassium iodide
E. Prednisone
Correct choice: E. Prednisone
Explanation: The associated image and described biopsy findings are consistent with the diagnosis of Sweet’s syndrome (aka acute febrile neutrophilic dermatosis), which typically presents with tender, erythematous edematous papules and plaques over the face and upper extremities. A vesiculobullous or pustular presentation is frequently associated with underlying myelogenous leukemia. Systemic steroids (e.g. prednisone), typically for 4-6 weeks, is the first line treatment for Sweet’s syndrome barring any contraindications. Furthermore, an excellent response to systemic steroids is one of the minor diagnostic criteria for Sweet’s syndrome. Potassium iodide, dapsone, and colchicine are considered 2nd line agents in the treatment of Sweet’s syndrome. Therehave been rare case reports of thalidomide being effective for recalcitrant Sweet’s syndrome.
82- A 54-year-old female presents to clinic with a bilateral eruption of red-brown papules on her dorsal hands and elbows. Some of the lesions are occasionally pruritic with a burning sensation. A biopsy is done and demonstates findings consistent with erythema elevatum diutinum (EED). Which of the following infections is EED associated with?
A. HSV
B. HIV
C. HPV
D. Leprosy
E. MRSA
Correct choice: B. HIV
Explanation: EED has been linked to a number of systemic diseases. Infections reported to occur in association with EED have included HIV, beta-hemolytic strep, hepatitis and tuberculosis. A literature review in 2012 identified 19 reported cases of EED in association with HIV. Nodular lesions progressing to bulky masses appear to be more common in EED patients with HIV. The relationship between EED and HIV is thought to involve immune complex deposition in blood vessels triggered by the HIV infection acting as antigenic stimuli. EED also has been associated with hematologic disorders, particularly IgA monoclonal gammopathy. Additionally, EED has been linked to a variety of autoimmune diseases, such as RA, IBD and lupus.
83- Periorificial dermatitis is thought to be a variant of rosacea. What is a known association with periorificial dermatitis?
A. Doxycycline intake
B. Gluten sensitivity
C. Sunscreen
D. Application of topical calcineurin inhibitors
E. Application of fluorinated topical steroids
Correct choice: E. Application of fluorinated topical steroids
Explanation: Periorificial dermatitis (or perioral dermatitis) has been associated with prior use of topical or inhaled fluorinated steroids, therefore they should be discontinued.
The other choices have not been associated with periorificial dermatitis. Doxycycline is a treatment for periorifacial dermatitis.
84- A 20-year-old female presents with prickling, tingling, burning and a stinging sensation within 30 minutes of water contact, which lasts for up to 2 hours. The symptoms begin on the lower extremities and then generalize, with sparing of the head, palms, soles and mucosa. She notes that the symptoms occur irrespective of the water temperature or salinity. On examination, specific skin lesions are not seen. What is the most likely diagnosis?
A. Polycythemia vera
B. Mastocytosis
C. Hypereosinophilic syndrome
D. Aquagenic pruritus
E. Aquagenic urticaria
Correct choice: D. Aquagenic pruritus
Explanation: Aquagenic pruritus is usually secondary to a systemic disease or another skin disorder such as urticaria or dermatographism. Primary idiopathic aquagenic pruritus is uncommon. Aquagenic pruritus presents with prickling, tingling, burning, or stinging sensations within 30 minutes of water contact, irrespective of its temperature or salinity, and lasts for up to 2 hours. Typically, symptoms begin on the lower extremities and then generalize, with sparing of the head, palms, soles, and mucosae. On exam, specific skin lesions are not seen. The pathologic mechanism is unknown although elevated dermal and epidermal level sof acetylcholine, histamine, serotonin, and prostaglandin E2 have been described.
1 - Polycythemia vera: Aquagenic pruritus can sometimes be secondary to a systemic disease such as polycythemia vera. However, the clinical scenario is a classic description of aquagenic pruritus, not polycythemia vera which presents with a ruddy complexion.
2 - Mastocytosis: Mastocytosis usually presents with cutaneous lesions and a positive Darier’s sign. In patients with aquagenic pruritus, specific skin lesions are not seen.
3 - Hypereosinophilic syndrome: Cutaneous lesions are seen in > 50% of patients with hypereosinophilic syndrome. Hypereosinophilic syndrome is on the differential diagnosis of pruritus or prickling sensation provoked by water contact. In patients with aquagenic pruritus, specific skin lesions are not seen.
5 - Aquagenic urticaria: Aquagenic pruritus can sometimes be secondary to another skin disorder such as urticaria or dermatographism. However, the clinical scenario is a classic description of aquagenic pruritus, not aquagenic urticaria. In aquagenic pruritus, specific skin lesions are not seen.
85- A 73-year-old man presented with a slow-healing postsurgical defect on the left cheek after undergoing Mohs surgery two months earlier. He has noticed a large amount of malodorous drainage from the site that has soaked through several bandages. What is the best dressing to use on this patient’s wound?
A. Hydrocolloid dressing
B. Hydrogel dressing
C. Alginate dressing
D. Film
E. Foam
Correct choice: C. Alginate dressing
Explanation: The correct answer is alginate dressings (Choice 3) which are polysaccharides derived from kelp and algae that are ideal for highly exudative wounds. Other advantages of alginates are that they confer hemostatic benefits and are suitable for use in sinuses. Hydrocolloid dressings (Choice 1), which are waterproof gel or foam within polyurethane films, are not suitable for this patient as these dressings are good for mildly exudative wounds. Hydrogel dressings (Choice 2), hydrophilic polymer holding significant amount of water, are not suitable for this patient as these dressings are best for dry, necrotic wounds. Films (Choice 4), thin layers of elastic polyurethane, provide a barrier against bacteria and are not the best choice for highly exudative wounds. Foams (Choice 5), hydrophobic polyurethane sheets with hydrophilic surface, are ideal for mildly to moderately exudative wounds.
86- A 17-year-old male has cystic acne vulgaris that is severe on his face, chest, and back x 3 years that is causing scarring. He has tried topical clindamycin and tretinoin 0.05% gel QAM and QPM, respectively, and washes with Neutrogena original. He is otherwise healthy. What is the next best step?
A. Start doxycycline 100 mg PO BID with food and water
B. Start isotretinoin weight-based dosing after checking baseline labs and enrolling in iPledge
C. Incise and drain each cyst
D. Begin a sulfur/sulfacetamide wash in place of Neutrogena wash
E. Start blue light therapy monthly for patient
Correct choice: B. Start isotretinoin weight-based dosing after checking baseline labs and enrolling in iPledge
Explanation: With a patient who has failed topical treatment who has severe acne burden with scarring, the correct choice is to start isotretinoin after enrolling in iPledge and checking baseline labs. With severe acne with scarring in a male patient who has failed topical treatment, the next most appropriate step is to initiate isotretinoin after enrolling in iPledge and checking baseline labs.
87- Which of the following carcinomas has been most associated with erythema gyratum repens?
A. Lung cancer
B. Breast cancer
C. Colon cancer
D. Prostate cancer
E. Upper GI tract cancer
Correct choice: A. Lung cancer
Explanation: 80% of cases of erythema gyratum repens have been associated with underlying malignancy. Lung cancer is the most common neoplasm. The skin eruption preceedes the detection of malignancy by an average of 9 months
88- Mastocytosis is a systemic disease that causes wheals upon rubbing a lesion (Darier’s sign). Mastocytosis can be measured by examination of serum:
A. Tryptase
B. Kinase
C. Bilirubin
D. Histamine
E. IgE
Correct choice: A. Tryptase
Explanation: The most common form of cutaneous mastocytosis is urticaria pigmentosa. It is diagnosed by measuring serum tryptase levels. The new diagnostic method is measurement of tryptase in bone-marrow blood, which is a new, sensitive marker of the mast cell burden in bone marrow of patients with systemic mastocytosis. The other markers do not diagnose mastocytosis.
89- Patients with dermatitis herpetiformis should avoid which of the following?
A. Corn
B. Rice
C. Oats
D. Sulfites
E. Iodides
Correct choice: E. Iodides
Explanation: Exposure to iodides has been reported to cause flares of dermatitis herpetiformis (DH).
This question assesses the examinee’s knowledge of triggers to be avoided in DH. Oral as well as topical exposure to iodides (choice 5) has been reported to cause flares of DH. Patients must strictly avoid consumption of gluten, which is found in wheat, barley, and rye; safe dietary grains include corn (choice 1), rice (choice 2), and oats (choice 3). Sulfites (choice 4) do not cause flares of DH.
90- A 42-year-old patient develops an abrupt and short-lived swelling of her tongue, mouth, and mucous membranes. There are no other parts of the body affected. The swelling is most pronounced around the eyes and lips. The internal lining of the upper respiratory tract is involved and the patient is immediately intubated. You suspect a diagnosis of angioedema. Which of the following medications the most likely cause of this reaction?
A. Lisinopril
B. Vancomycin
C. Ampicillin
D. Trimethoprim-sulfamethoxazole
E. Doxycycline
Correct choice: A. Lisinopril
Explanation: A number of drugs can cause angioedema without wheals. The most common are NSAIDs and ACE inhibitors. Aspirin intolerance may present with angioedema alone, or with urticaria or anaphylaxis; cross-reactions with other NSAIDs can occur. A causative link to ACE inhibitors may be overlooked because angioedema can present over a year after starting the medication. For practical purposes, ACE inhibitors should be discontinued in favor of other antihypertensive medications if angioedema occurs without wheals. ACE inhibitors are contraindicated in patients with HAE and acquired C1 inh deficiency.
2-Vancomycin is not the most common cause of angioedema out of the medications listed. 3-Ampicillin is not the most common cause of angioedema out of the medications listed.
4-Trimethoprim-sulfamethoxazole is not the most common cause of angioedema out of the medications listed.
5-Doxycycline is not the most common cause of angioedema out of the medications listed.
91- An 8-year-old boy presents with subcutaneous nodules over bony prominences, migratory joint pains, chest pain, and rapid, irregular, and aimless involuntary movements of the arms and legs, trunk and facial muscles. On closer examination, you notice that he has an evanescent annular and polycyclic erythematous eruption. You suspect he may have cutaneous and clinical manifestations of acute rheumatic fever. What is the most appropriate diagnosis for his cutaneous eruption?
A. Annular erythema of infancy
B. Erythema migrans
C. Erythema marginatum
D. Erythema gyratum repens
E. Erythema annulare centrifugum
Correct choice: C. Erythema marginatum
Explanation: Erythema marginatum: Erythema marginatum rheumaticum is a migratory, annular and polycylic erythematous eruption. It is a cutaneous manifestation of acute rheumatic fever. Associated findings incude: carditis, migratory polyarthritis, Sydenham chorea, and subcutaneous nodules. It is seen more commonly in children than in adults.
1 - Annular erythema of infancy: The differential diagnosis of erythema marginatum inclues annular erythema of infancy however the patient is not an infant in this scenario. The typical associated findings of acute rheumatic fever are absent in this condition.
2 - Erythema migrans: Erythema migrans is an annular erythema that develops at the site of a bite of a Borrelia-infected tick. Several species of Ixodes ticks are infected with different genospecies of Borrelia burgdorferi. Erythema migrans represents the initial cutanoues manifestation of Lyme disease and is seen in 60-90% of patients diagnosed with the disease. Multiple secondary lesions that are smaller in size can occur as a result of spirochetemia or lymphatic spread.
4 - Erythema gyratum repens: Erythema gyratum repens is a figurate erythema that is migratory and copmosed of concentric rings with a wood-grain appearance. It represents a paraneoplastic phenomenon and the most common underlying neoplasm is carcinoma of the lung. The lesions may have associated pruritus and scale, and they characteristically exhibit rapid migration (up to 1 cm per day). The cutaneous lesions resolve when the neoplasm is successfully treated.
5 - Erythema annulare centrifugum: Erythema annulare centrifugum is characterized by erythematous annular lesions that migrate centrifugally. Superficial lesions can have the classic trailing white scale, while the deep gyrate erythemas have a more infiltrated border. The disorder occurs more commonly in adults, and superficial lesions favor the thighs and hips. Individual lesions usually last for several days to a few months. Although often idiopathic in nature, it can be associated with infecitons (i.e. tinea pedis) and anecdotally with other disorders or exposures.
92- A patient in your occupational dermatology clinic presents with mottled hyperpigmentation with areas of hypopigmentation and hair loss. She has a history of numerous SCCis (squamous cell carcinomas in situ). Upon further examination, she has many symmetric, yellow, punctate corn-like papules on both palms (shown in image). The patient likely was chronically exposed to which of the following?
A. Mercury
B. Lead
C. Arsenic
D. Bismuth
E. Chromium
Correct choice: C. Arsenic
Explanation: Chronic arsenic exposure can occur through contaminated drinking water or occupational exposure. It is characterized by mottled hyperpigmentation with areas of hypopigmentation, keratoses on the palms and soles and other sites, alopecia, and multiple non- melanoma skin cancers, particularly Bowen’s disease (SCCis). Arsenical keratoses are
precancerous papules on the palms and soles that are symmetric, yellow and punctate. They measure 2-10 mm in size and favor the thenar and hypothenar eminences, distal palms, lateral fingers, dorsal aspect of the interphalangeal joints, and weight-bearing plantar surfaces. They rarely occur on the trunk, proximal extremities, eyelids and genitalia and may coalesce into plaques.
1 – Mercury poisoning and exposure may present with acrodynia, tattoo reaction (cinnabar), granulomas, exanthema, slate-gray discoloration, allergic and irritant contact dermatitis and baboon syndrome. 2 – Lead poisoning may present with irritant contact dermatitis and a gingival lead line.
4 – Exposure to bismuth may present with irritant contact dermatitis and black tongue. 5 – Chromium exposure may present with allergic and irritant contact dermatitis, skin ulceration, and burns. Systemic absorption can lead to renal failure, hepatic failure, anemia, and coagulopathy.
93- Autoantibodies to ECM-1 may be associated with which of the following phenotypic alterations?
A. Hoarse cry at birth, with beaded papules at the eyelid margins noted later in life
B. 1-3mm yellow papules and wrinkled skin of the lateral neck and flexures
C. Thickened toenails with painful plantar keratoderma
D. Tense blisters arising within urticarial plaques
E. Pruritic, atrophic scarring of the genitalia
Correct choice: E. Pruritic, atrophic scarring of the genitalia
Explanation: Antibodies to ECM-1 are associated with lichen sclerosus, choice 5. Lichen sclerosus is characterized by pruritic, atrophic plaques of the vulva, though this condition can affect the male genitals or extragenital skin as well. Choice 1 describes Lipoid Proteinosis, which is caused by a mutation in ECM-1, as opposed to antibodies. Choice 2 described Pseudoxanthoma elasticum, which is caused by a mutation in ABCC6. Choice 3 describes Pachyonychia Congenita, a family of diseases associated with mutations in keratins 6 and 16. Choice 4 describes bullous pemphigoid (or a BP-like presentation of EBA), which is caused by antibodies to BPAG1 or 2 (or Collagen 7 in the case of EBA).
94- You are called to see a consult in the emergency room. The patient has erythema and scaling involving 90% of the body surface area, peripheral edema, tachycardia, and a fever. From an epidemiological standpoint, which of the following is the most common cause of erythroderma?
A. Idiopathic
B. Atopic dermatitis
C. Cutaneous T-cell lymphoma
D. Psoriasis
E. Drug reactions
Correct choice: A. Idiopathic
Explanation: The most common causes of erythroderma are idiopathic, dermatitis, psoriasis, drug reactions, and cutaneous T-cell lymphoma. In a study of 746 patients, the underlying causes of erythroderma were dermatitis (24%), psoriasis (20%), drug reactions (19%), and CTCL (8%). When categories within the dermatitis group were examined, atopic dermatitis (9%) was the most common type, followed by contact dermatitis (6%), seborrheic dermatitis (4%), and chronic actinic dermatitis (3%). Despite multiple skin biopsies, in-depth clinical investigations, and detailed medical histories, the underlying cause of erythroderma cannot be found in 25% of patients. Cases of idiopathic erythroderma also tend to be chronic and are more likely to recur after treatment.
2 – According to a study of 746 patients, dermatitis represents 24% of the most common underlying causes of erythroderma. When categories of the dermatitis group were examined, atopic dermatitis (9%) was the most common type, followed by contact dermatitis (6%), seborrheic dermatitis (4%), and chronic actinic dermatitis (3%). 3 – Cutaneous T-cell lymphoma represents about 8% of the most common underlying causes of erythroderma. 4 – Psoriasis represents about 20% of the most common underlying causes of erythroderma. 5 – Drug reactions represent approximately 19% of the most common underlying causes of erythroderma.
95- A mother brings here 3-year-old child in for an intensely pruritic rash. What is the next step in management?
A. Potassium hydroxide prep
B. Wound culture
C. Tzanck smear
D. Scabies prep
E. Punch biopsy
Correct choice: D. Scabies prep
Explanation: The diagnosis is scabies and therefore a scabies prep is the appropriate next step in management. Characteristic features include burrows and crusted papules in the webbing between digits. Other commonly involved areas include the areolae, umbilicus and in particular the penis and scrotum where characteristic nodules may form. In infants, the head may be involved, which is rarely seen in adult patients. A scabies prep can be performed using a #15 blade with a drop of mineral oil followed by evaluation under the microscope. A wound culture is used to evaluate for bacterial or candidal infection. Pustules should be unroofed and the purulent contents swabbed. A tzanck smear is performed to evaluate herpetic infections. The scrapings from the base of an unroofed blister reveal multinucleated giant cells. A potassium hydroxide prep is performed when there is clinical suspicion of a fungal infection. Most often, the rash will be annular with a leading
scale. A punch biopsy is performed when the diagnosis is in question. A punch biopsy is selected when the suspected pathology involves the deeper skin structures. Though a biopsy could confirm the diagnosis of scabies, a scabies prep is an in office or bedside procedure offering rapid diagnosis.
96- Nail findings occur in 10-15% of cases of lichen planus, usually in combination with other classic cutaneous lesions. Which of the following is not a commonly reported feature of nail lichen planus?
A. Trachyonychia
B. Onychoschizia
C. Anonychia
D. Dorsal pterygium
E. Ventral pterygium
Correct choice: E. Ventral pterygium
Explanation: Lichen planus of the nails occurs in 10-15% of cases, usually in combination with other LP lesions on the skin. 20-nail dystrophy (trachyonychia) can be seen, but it is not diagnostic of LP. Thinning, longitudinal ridging, and distal splitting of the nail plate can occur (onychoschizia). Also, onycholysis, longitudinal striation (onychorrhexis), subungual hyperkeratosis, or anonychia can be seen. The classic finding is dorsal pterygium or forward growth of the eponychium with adherence of the proximal nail plate. Ventral pterygium is more classicaly seen in Scleroderma.
97- A 19-year-old female presents with chronic, focal intense pruritus of the upper back, especially over the right medial scapular border. She has had these symptoms since childhood.
On exam you note that this corresponds to the area innervated by the posterior rami of thoracic spinal nerves 2-6. Occasionally she senses paresthesias. On exam, you notice a hyperpigmented patch due to chronic rubbing of the affected area. Given the onset during childhood, this condition may also represent a sign of which syndrome?
A. Multiple endocrine neoplasia type 1
B. Multiple endocrine neoplasia type 2A
C. Multiple endocrine neoplasia type 2B
D. Tuberous sclerosis
E. Neurofibromatosis
Correct choice: B. Multiple endocrine neoplasia type 2A
Explanation: Multiple endocrine neoplasia type 2A: Notalgia paresthetica is a common condition in adults that is characterized by focal, intense pruritus of the upper back, especially over the medial scapular borders; this corresponds to the area innervated by the posterior rami of thoracic spinal nerves 2–6 (T2–T6). Occasionally, patients also report pain, paresthesias, or hyperesthesias. A characteristic finding on physical examination is a hyperpigmented patch due to chronic rubbing of the affected area. Dermal melanophages are evident histologically, and there is considerable overlap with macular amyloidosis.Most evidence suggests that notalgia paresthetica represents a sensory neuropathy. Degenerative changes of the spine corresponding to the affected dermatome(s) are evident in ~60% of affected patients, suggesting a pathogenic role for spinal nerve impingement. The posterior rami of T2–T6 pursue a right-angle course through the multifidus spinae muscle, potentially predisposing them to entrapment and injury. Notalgia paresthetica may also represent a sign of multiple endocrine neoplasia type 2A (Sipple syndrome), especially if the onset is during childhood or adolescence.
1 - Multiple endocrine neoplasia type 1:
Notalgia paresthetica may also represent a sign of multiple endocrine neoplasia type 2A (Sipple syndrome), especially if the onset is during childhood or adolescence. It is not associated with multiple endrocine neoplasia type 1.
3 - Multiple endocrine neoplasia type 2B:
Notalgia paresthetica may also represent a sign of multiple endocrine neoplasia type 2A (Sipple syndrome), especially if the onset is during childhood or adolescence. It is not associated with multiple endrocine neoplasia type 2B.
4 - Tuberous sclerosis: Notalgia paresthetica may also represent a sign of multiple endocrine neoplasia type 2A (Sipple syndrome), especially if the onset is during childhood or adolescence. It is not associated with tuberous sclerosis.
5 - Neurofibromatosis: Notalgia paresthetica may also represent a sign of multiple endocrine neoplasia type 2A (Sipple syndrome), especially if the onset is during childhood or adolescence. It is not associated with neurofibromatosis.
98- A 38-year-old African American female presents with grouped follicular papules, acneiform lesions, indurated plaques and tumors which are most pronounced in the head and neck area (see image). The skin lesions are associated with alopecia and mucinorrhea. She has infiltrated plaques in both eyebrows and complains of severe pruritus. She has experienced frequent secondary bacterial infections with Staphylococcus aureus. Which of the following is the most common phenotype of this patient’s condition?
A. CD3+, CD4+, CD8-
B. CD3+, CD4-, CD8-
C. CD3-, CD4+, CD8+
D. CD3-, CD4-, CD8+
E. CD3+, CD4-, CD8+
Correct choice: A. CD3+, CD4+, CD8-
Explanation: Folliculotropic mycosis fungoides is a distinct variant of mycosis fungoides characterized by the presence of folliculotropic infiltrates with preferential involvement of the head and neck region. Patients may present with grouped follicular papules, acneiform lesions, indurated plaques and sometimes tumors. Skin lesions are often associated with alopecia and sometimes mucinorrhea. Infiltrated plaques in the eyebrows and concurrent alopecia are common and highly characteristic. In most cases, the neoplastic T cells have a CD3+, CD4+, CD8- phenotype as in classic mycosis fungoides. Admixture with CD30-positive blast cells is common.
2 – In most cases of folliculotropic mycosis fungoides, the neoplastic T cells have a CD3+, CD4+, CD8- phenotype as in classic mycosis fungoides. Admixture with CD30-positive blast cells is common. 3 – In most cases of folliculotropic mycosis fungoides, the neoplastic T cells have a CD3+, CD4+, CD8- phenotype as in classic mycosis fungoides. Admixture with CD30-positive blast cells is common. 4 – In most cases of folliculotropic mycosis fungoides, the neoplastic T
cells have a CD3+, CD4+, CD8- phenotype as in classic mycosis fungoides. Admixture with CD30- positive blast cells is common. 5 - In most cases of folliculotropic mycosis fungoides, the neoplastic T cells have a CD3+, CD4+, CD8- phenotype as in classic mycosis fungoides. Admixture with CD30-positive blast cells is common.
99- Which of the following is the most likely diagnosis?
A. Erythema multiforme
B. Annular urticaria
C. Secondary syphilis
D. Fixed drug eruption
E. Polymorphous light eruption
Correct choice: A. Erythema multiforme
Explanation:Erythema multiforme (EM) presents with target lesions favoring the extensor extremities.
Without a history, the examinee must discover distinguishing features in the clinical photo to arrive at the correct diagnosis. On the dorsal hand, there are variable edematous urticarial papules, some of which feature three zones: a dusky center surrounded by two concentric rings of different colors. Although not all of the lesions are fully developed, based on the presence of typical targets, EM (choice 1) is the best answer. The differential diagnosis includes annular urticaria (choice 2), which displays central erythema or normal skin, never epidermal damage as evidence by duskiness, necrosis, or vesiculation; secondary syphilis (choice 3), which tends to favor the palms over the dorsa and usually has scale; fixed drug eruption (choice 4), which can be morphologically similar to
EM but generally shows fewer lesions; and polymorphous light eruption (choice 5), which also presents on the sun-exposed extremities and may at times be difficult to distinguish from EM, but will not have true targets. In clinical practice, additional history and possibly biopsy will readily differentiate between these entities.
100- A 55-year-old female presents with a one month history of the following clinical pictures. She additionally has developed thickened nails with subungal debris and a few small hyperkeratotic red plaques on the abdomen. What is the most likely diagnosis?
A. Mal de meleda syndrome
B. Pityriasis rubra pilaris
C. Psoriasis vulgaris
D. Lichen spinulosus
E. Cutaneous T-cell lymphoma
Correct choice: B. Pityriasis rubra pilaris
Explanation: This presentation of a cutaneous eruption of palmer keratoderma, hyperkeratotic follicular papules (“nutmeg grater”), red-orange plaques and nail findings is most consistent with Type I pityriasis rubra pilaris. This clinical presentation less likely represents the other answer choices.
101- What is the most likely diagnosis based on the picture?
A. Neurotic excoriations
B. Reactive perforating collagenosis
C. Disseminated herpes simplex virus infection
D. Miliaria crystallina
E. Pyoderma gangrenosum
Correct choice: B. Reactive perforating collagenosis
Explanation: The patient has reactive perforating collagenosis (RPC) characterized by collagen extruding through the skin. While it is not possible to discern which perforating disorder the patient has without biopsy and clinicopathologic correlation, the only perforating disorder option on the list is RPC. Neurotic excoriations is incorrect as excoriations are a secondary lesion and the patient has primary lesions (that are not excoriated). Disseminated or localized herpes simplex virus should present as vesicles, not papules with the appearance of central substance extrusion. Miliaria crystallina presents with translucent, small 1-2 mm vesicles that appear wet, which is not the appearance of the rash in the picture. Pyoderma gangrenosum is a neutrophil-rich inflammatory condition of the skin characterized by a non-healing ulcer with a red to purple appearance with an undermined border.
102- Which of the following is the most common nail finding in patients with alopecia areata?
A. Oil spots
B. Onycholysis
C. Splinter hemorrhages
D. Trachyonychia
E. Pitting
Correct choice: E. Pitting
Explanation: Nail changes are a common feature of alopecia areata (AA), with an average prevalence of 30%, and can cause significant disfigurement and loss of function. Pitting and trachyonychia were by far the most common manifestations of AA, with an average prevalence of 20 and 8%, respectively. Red spotted lunulae, onycholysis, and punctate leukonychia were other reported nail findings in AA. Other etiologies, such as onychomycosis or lichen planus, may coexist with or confound the diagnosis.
103- Which of the following conditions is associated with an IgM monoclonal gammopathy?
A. Schnitzler’s syndrome
B. Sweet’s syndrome
C. Scleredema
D. Scleromyxedema
E. Subcorneal pustular dermatosis
Correct choice: A. Schnitzler’s syndrome
Explanation: Schnitzler’s syndrome is associated with IgM monoclonal gammopathy.
This question tests the examinee’s knowledge of monoclonal gammopathies of note in dermatology. Of the answer choices, only Schnitzler’s syndrome (choice 1) is associated with IgM monoclonal gammopathy, in association with chronic urticaria, bone pain, and fevers. Sweet’s syndrome (choice 2) and subcorneal pustular dermatosis (choice 5) are associated with IgA gammopathy; scleredema (choice 3), with IgGκ; and scleromyxedema (choice 4), with IgGλ.
104- What is the appropriate and necessary medical management for a seborrheic keratosis diagnosed by biopsy?
A. Superficial radiation therapy
B. Electrodessication and curettage
C. Excision with 5 mm margin
D. Cryotherapy
E. Reassurance
Correct choice: E. Reassurance
Explanation: Seborrheic keratoses are benign lesions that do not require additional treatment. You may consider cryotherapy and/or curettage to seborrheic keratosis lesions that are bothersome for a patient but neither is necessary. Superficial radiation therapy is not appropriate treatment for a seborrheic keratosis. Excision with a 5 mm margin is an over-treatment for a seborrheic keratosis.
105- What is the diagnosis?
A. Granuloma annulare
B. Tinea corporis
C. Erythema annulare centrifigum
D. Subacute cutaneous lupus
E. Cutaneous larva migrans
Correct choice: A. Granuloma annulare
Explanation: Granuloma annulare is a benign condition characterized by annular dermal plaques. They are often pink to skin-colored. High potency topical or intralesional steroids may be used, though often observation and reassurance is the treatment of choice. Tinea corporis is a cutaneous dermatophyte with a characteristic annular appearance with an advancing edge. Erythema annulare centrifigum is a gyrate characterized by erythematous, annular patches and plaques with a trailing scale. This feature can help distinguish from tinea corporis. Subacute cutaneous lupus (SCLE) presents with annular and often scaly patches and plaques. It is most commonly seen in photodistributed areas. There is an association between SCLE and the anti-Ro antibody. Cutaneous larva migrans is a cutaneous hookworm infection acquired most commonly by walking barefoot on contaminated grounds. The classic serpiginous arrangement represents movement of the hookworm through the epidermis.
106- A patient presents to your clinic with Raynaud’s of the hands, and purpura on the lower extremities. Lab testing is notable for positive cryoglobulins, with the subtype revealing monoclonal IgM only. Which of the following underlying conditions is most likely?
A. Hepatitis B
B. Hepatitis C
C. Waldenström macroglobulinemia
D. Angioimmunoblastic T cell lymphoma
E. Systemic sclerosis
Correct choice: C. Waldenström macroglobulinemia
Explanation: The stem describes a case of Type I cryoglobulinemia, which is typified by monoclonal IgM or IgG (no rheumatoid factor activity). Type I cryoglobulins are always linked to a B-cell lymphoproliferative disorder, that is, multiple myeloma, Waldenström macroglobulinemia, chronic lymphocytic leukemia, B-cell non-Hodgkin lymphoma, or hairy cell leukemia. Mixed cryoglobulinemias are characterized by monoclonal IgM (or IgG) with polyclonal IgG (Type II) or polyclonal IgM complexed with polyclonal IgG (Type III). Mixed cryoglobulinemias (type II or III) are associated with hepatitis C virus (HCV) infection, systemic autoimmune diseases, lymphoproliferative disorders, and other chronic infections.
107- First line treatment for this condition includes:
A. Intralesional steroid injections
B. 2.4 million units IM benzathine penicillin
C. Methotrexate
D. Ruxolitinib
E. Treatment for underlying lymphoma
Correct choice: A. Intralesional steroid injections
Explanation: This image demonstrates exclamation point hairs of alopecia areata. Intralesional injection with low potency steroids is a first line treatment option.
108- A patient with scalp psoriasis has recently started a medication which results in increased levels of cyclic AMP which decreases production of TNF, IL-17, IL-23. You inform her that the most common side effect upon initially starting this medication is which of the following?
A. Skin site reaction
B. Diarrhea
C. Fungal infections
D. Reactivation of tuberculosis
E. Dizziness
Correct choice: B. Diarrhea
Explanation: Apremilast is a phosphodiesterase 4 inhibitor which results in increased levels of cyclic AMP which in turn decreases production of TNF, IL-17, IL-23. It is dosed twice daily and is titrated up over 6 days to a final dose of 30 mg PO twice daily to minimize diarrhea. It is relatively more effective for palmoplantar psoriasis and scalp psoriasis. The most common side effects are GI upset/diarrhea in 20% of cases.
109- A 40-year-old male presents with a chronic pruritic eruption. Which of the following is true?
A. Apple green birefringence on PAS stain
B. Improves after treating underlying infection
C. Stains PAS positive and diastase resistant
D. Demonstrates amyloid deposition predominantly around blood vessels
E. Associated with a lymphoproliferative disorder
Correct choice: C. Stains PAS positive and diastase resistant
Explanation: Choice 1 is actually apple green birefringensce on CONGO-RED staining; choice 2, there is no underlyin infection in lichen amyloid; choice 4, lichen amyloid demonstrates dermal amyloid deposition, not perivascular; choice 5 lichen amyloid is not associated with underlying malignancy (like primary systemic amyloid)
110- A 20-year-old female presents with an allergic contact dermatitis to a perfume containing Lily of the valley. What is the causative allergen?
A. Cinnamic acid
B. Vanillin
C. Hydroxycitronellal
D. Atranorin
E. Evernic acid
Correct choice: C. Hydroxycitronellal
Explanation: The main allergen in Lily of the valley, which is found in perfumes, soaps, cosmetics, eye cream and aftershaves, is hydroxycitronellal (synthetic). Cinnamic acid and vanillin are present in Balsam of Peru, while atranorin and evernic acid are allergens in oak moss absolute.
111- This 48 year old woman tells you these lesions began as “acne bumps” on her legs. A biopsy reveals ulceration and a mixed inflammatory infiltrate, including neutrophils. A tissue culture for infectious organisms is negative. What is the most likely underlying systemic condition in this patient?
A. Celiac disease
B. Ovarian cancer
C. Chronic pancreatitis
D. IgA monoclonal gammopathy
E. Castleman’s disease
Correct choice: D. IgA monoclonal gammopathy
Explanation: The associated image and described biopsy findings are consistent with a diagnosis of pyoderma gangrenosum (PG), which most often presents as an acneiform papule or pustule that progresses into a rapidly expanding ulceration with an undermined border. ~50% of patients with PG have an underlying systemic condition, most commonly inflammatory bowel disease (ulcerative colitis more often than Crohn’s disease). Other associated conditions include: arthritis (typically asymmetric, seronegative, monoarticular arthritis of the large joints), leukemia (usually myelogenous), multiple myeloma, IgA monoclonal gammopathy, polycythemia, Hepatitis C infection, HIV infection, and systemic lupus erythematosus. Celiac disease is associated with dermatitis herpetiformis. Ovarian cancer may underlie the development of dermatomyositis. Chronic pancreatitis is associated with pancreatic panniculitis. Lastly, paraneoplastic pemphigus may develop in patients with Castleman’s disease.
112- A 19-year-old male presents with asymptomatic, multiple, yellow-white papules on the vermilion portion of the upper lip. The papules are no more than 1-2 mm in diameter and are present bilaterally and also on the buccal mucosa. The patient recently underwent puberty, and is otherwise healthy. Which of the following is the most appropriate term for these ectopic sebaceous glands in this location?
A. Montgomery tubercles
B. Fordyce spots
C. Meibomian glands
D. Glands of Zeis
E. Tyson glands
Correct choice: B. Fordyce spots
Explanation: Fordyce granules are sebaceous glands that are located in the vermilion zone of the lips and the oral mucosa. Historically, these sebaceous glands have been considered ectopic; however, because they are such a common clinical finding in the oral cavity, Fordyce granules should be regarded as a variation of normal anatomy. “Free” sebaceous glands (see below) can also be found on the eyelids (meibomian glands), areolae (Montgomery tubercles), labia minora, and prepuce (Tyson glands).
1- Montgomery tubercles are ectopic sebaceous glands found around the areolae.
3- Meibomian glands are ectopic sebaceous glands found on the eyelids.
4- Glands of Zeis are unilobar sebaceous glands located on the margin of the eyelid. The glands of Zeis service the eyelash. These glands produce an oily substance that is issued through the excretory ducts of the sebaceous lobule into the middle portion of the hair follicle.
5- Tyson glands are ectopic sebaceous glands found on the prepuce.
113- You diagnose a 22-year-old male member of the army with tinea versicolor of the back, chest, and arms. The rash flares in the summertime. What is the most appropriate initial recommendation?
A. Fluconazole 200 mg by mouth daily ongoing
B. Selenium sulfide 1% lotion daily to affected areas
C. Frequent moisturizers
D. BenzaClin (benzoyl peroxide/clindamycin) 5%/1% daily to affected areas
E. Hydrogen peroxide to the involved areas
Correct choice: B. Selenium sulfide 1% lotion daily to affected areas
Explanation: The patient has tinea versicolor and should be treated with selenium sulfide as the starting therapy. You may consider fluconazole by mouth intermittent dosing for severe, refractory cases of tinea versicolor but you should not use ongoing fluconazole therapy as treatment for the condition. Benzoyl peroxide/clindamycin is used for acne. Hydrogen peroxide solutions are used to treat seborrheic keratoses. Frequent moisturizers will not treat tinea versicolor.
114- The best diagnosis for this congenital pale patch without any extracutaneous associations is:
A. Hypomelanosis of Ito
B. Segmental vitiligo
C. Ash leaf spots
D. Nevus anemicus
E. Nevus depigmentosus
Correct choice: E. Nevus depigmentosus
Explanation: Nevus achromicus is interchangeable for nevus depigmentosus. It usually presents at birth or appear during early infancy as normal pigmentation increases. Most individuals will have a solitary lesion of nevus depigmentosus, but multiple lesions and segmental forms of nevus depigmentosus have been described.
Nevus depigmentosus tends to persist lifelong, but remains unchanged after onset. The hypopigmented white spots of tuberous sclerosis are most difficult to distinguish from nevus depigmentosus, but lack of other cutaneous or systemic manifestaions exclude tubrous sclerosis. Lesions of vitiligo tend to be depigmented (melanocytopenic not melanopenic as in the question), and show a bright white coloration with Wood’s lamp examination. Nevus anemicus is a distinct vascular birthmark characterized by blanching of cutaneous blood vessels, hence presenting as a “white” patch of skin that becomes unnoticeable when the surrounding skin is blanched with a glass slide (“diascopy”).
Achromic nevus is an uncommon birthmark characterized by a well-defined pale patch. This is usually several centimeters in diameter, with an irregular but well-defined border. Shape and size varies. Often, smaller hypopigmented macules arise around the edges, resembling a splash of paint. Achromic nevus is also known as nevus depigmentosus and non-pigmented nevus. The name is not quite right, as thehypomelanotic patches of an achromic nevus are not completely white, unlike the areas of depigmentation in vitiligo, which are amelanotic, and completely lacking melanin. Achromic nevi are usually solitary, in contrast to tuberous sclerosis, where multiple pale patches occur and are called ash-leaf spots. Achromic nevus is usually noted at birth or early childhood, although lesions may not be apparent until mid-childhood in those with light-colored skin. The nevi remain stable over time. Achromic nevus most commonly arises on the trunk, but may also arise on the limbs and elsewhere.
115- Which extracutaneous organ is typically associated with this subtype of sarcoidosis?
A. Heart
B. Lungs
C. Kidneys
D. Eyes
E. Liver
Correct choice: B. Lungs
Explanation: This is an example of lupus pernio, which is often associated with chronic sarcoidosis of the lungs. Although the patient may have involvement of other organs, pulmonary sarcoid is most often seen with lupus pernio.
116- An 87-year-old female gets a dry flaky rash on her lower legs and arms each wintertime when the humidity decreases. The areas are itchy. She scrubs her arms and legs very hard to clean them. What is the most appropriate treatment for this patient?
A. Topical emollients at least three times per day and gentle skin care
B. Clotrimazole 1% cream two times a day to affected areas of the skin
C. Unna wrap the arms and legs weekly
D. Clobetasol 0.05% ointment three times a day to affected areas of the skin
E. Fluconazole 200 mg by mouth weekly x 4 weeks then stop
Correct choice: A. Topical emollients at least three times per day and gentle skin care
Explanation: The correct answer is gentle skin care and the use of emollients for the patient described who has winter’s itch, otherwise known as asteatotic eczema. Topical anti-fungal and systemic anti-fungal medications are not indicated for this condition. You may use topical steroid in the future if not responsive to emollient and gentle skin care but you would start with a lower strength steroid than clobetasol. Unna wraps are helpful for individuals who are picking their skin but are not indicated for the patient in this vignette.
117- This patient presents with a papule on the abdomen that he would like you to remove. He states he’s always had it and finds it to be unattractive. Which of the following is true?
A. A similar finding is present in 20% of the general population
B. In women there is no change with changing hormones
C. On histology demonstrate smooth muscle
D. Has increased malignant potential and should be immediatly removed
E. Renal imaging is recommended
Correct choice: C. On histology demonstrate smooth muscle
Explanation: The patient has a supernumerary nipple. Found in 1-6% of the general population. Histology shows increased smooth muscle, sebaceous glands that are opened directly onto skin surface, and mammary glands. There is no increased malignant potential but examination such as mammography or ultrasound has been recommended as with other breast tissue. Routine renal imaging is not recommended. Can be present in Incontinentia Pigmenti patients.
118- Compared to children, adults with the condition pictured are more likely to develop which of the following complications?
A. Abdominal pain
B. Persistent fever
C. Hepatic impairment
D. Renal impairment
E. Cerebral arteriovenous malformation
Correct choice: D. Renal impairment
Explanation: Adult patients with Henoch-Schönlein purpura (HSP) are more likely than children to develop persistent renal impairment.
This question tests the examinee’s ability to recognize HSP presenting as palpable purpura on the lower extremity and knowledge of how HSP complications differ in adults versus children. Compared to children, adults with HSP are at higher risk for renal impairment (choice 4), but less
commonly experience abdominal pain (choice 1) or fever (choice 2). Hepatic impairment (choice 3) and cerebral arteriovenous malformations (choice 5) are not commonly recognized sequelae of HSP.
119- A 42 year-old woman has the findings pictured, as well as erythematous patches on the upper chest and shoulders. No Raynaud’s, polyarthritis, pulmonary, or cardiac involvement is noted. Which antibody might you expect to be present in this patient?
A. Anti-SRB
B. Anti-Jo-1
C. Anti-Mi-2
D. Anti-Ku
E. Anti-La
Correct choice: C. Anti-Mi-2
Explanation: This patient has dermatomyositis. Anti-Mi-2 antibodies in DM correlate with the presence of a shawl sign, cuticular changes, and good prognosis. Anti-Jo-1 antibodies correlate with pulmonary fibrosis, Raynaud’s, and polyarthritis. Anti-SRP antibodies correlate with cardiac disease and poor prognosis. Anti-Ku antibodies correlate with sclerodermatomyositis. Anti-La antibodies correlate with Sjogren’s syndrome.
120 - The syndrome that is associated with the disease shown in this image is known as:
A. SAPHO syndrome
B. Follicular occlusion tetrad syndrome
C. LEOPARD syndrome
D. NAME syndrome
E. PAPA syndrome
Correct choice: E. PAPA syndrome
Explanation: PAPA syndrome ( Pyogenic Arthritis, Pyoderma gangerosum and Acne) The image shows classic ulcerative pyoderma gangrenosum with undermined violaceous gray border. Re- epithelialization occurs from the margins and the ulcer heal usually with atrophic cribriform pigmented scar. Although the classic morphologic clinical presentation of pyoderma gangrenosum is an ulceration, there are several variants (bullous, pustular, and superficial granulomatous) which differ by their clinical presentation, location, and associated diseases.
121- A patient with plaque psoriasis complains of morning stiffness of joints lasting longer than 60 minutes. On exam, you notice nail involvement and reduced range of motion of her shoulders, neck, and lower back. You order an x-ray for further evaluation, to see if this patient has psoriatic arthritis. Which of the following radiographic features would support a diagnosis of psoriatic arthritis?
A. Osteophyte formation
B. Tuft resorption
C. Subchondral sclerosis
D. Cyst formation
E. Bony enlargement
Correct choice: B. Tuft resorption
Explanation: Radiographic features of psoriatic arthritis include “sausage digits;” large eccentric erosions; pencil-in-cup deformities – erosive changes of the joint – phalangeal distal tip is pencil on
an eroded cuplike joint space; tuft resorption – acroosteolysis; periostitis – inflammation of the periosteum; and sacroileitis. The other changes are seen on radiograph in osteoarthritis.
122- This is a child’s finger, what may be associated with this condition?
A. Osteoma cutis
B. Diabetes
C. Lupus
D. Calcinosis cutis
E. Cryoglobulins
Correct choice: E. Cryoglobulins
Explanation: In kids, pernio can be associated with cryoglobulins and cold agglutinins. It is triggered by cold and wet which results in acral violaceous color with burning and itching.
123- Which of the following conditions can predispose the patient to warfarin-induced skin necrosis?
A. Factor V Leiden
B. Protein C deficiency
C. Protein S deficiency
D. Antithrombin III deficiency
E. All of the above
Correct choice: E. All of the above
Explanation: All of these conditions are pro-coagulopathic and therefore predispose the patient to warfarin-induced skin necrosis.
124- What is the most common drug offender in drug-induced bullous pemphigoid?
A. Penicillin
B. Naproxen
C. Propranolol
D. Furosemide
E. Spironolactone
Correct choice: D. Furosemide
Explanation: Furosemide is the most common cause of druginduced BP. Sulfasalazine is another common offender.
125- A patient with renal amyloidosis, urticaria, fevers, limb pains, and deafness is diagnosed with Muckle-Wells Syndrome. What is the mutation?
A. MEFV
B. MVK
C. TNF-1 Receptor
D. CIAS1
E. CD2BP1
Correct choice: D. CIAS1
Explanation: Patients with Muckle-Wells has amyloid AA fibrils. It is an autosomal dominant condition characterized by deafness, hives, and mutation in the CIAS1 gene
126- A 48-year-old white woman presents to your office complaining of a new rash and recent-onset difficulty walking up and down stairs. On exam you notice peri-ungual erythema of the hands, ragged fingernail cuticles, and poikiloderma on her upper chest and back. Which of the following would be most important to include in her work up for malignancy?
A. Mammogram
B. Chest X-ray
C. Thyroid ultrasound
D. Transvaginal ultrasound
E. Colonoscopy
Correct choice: D. Transvaginal ultrasound
Explanation: Dermatomyositis is an idiopathic inflammatory disease characterized by myositis and typical cutaneous manifestations. There is an increased incidence of malignancy in these patients which may precede, occur with, or follow the diagnosis of dermatomyositis. The most common form of malignancy in adult white women with dermatomyositis is ovarian cancer, thus a transvaginal ultrasound is the best answer as this is a common screening test for ovarian cancer.
Other malignancies that have been associated with dermatomyositis include testicular cancer, gastrointestinal, lung and nasopharyngeal carcinomas. Although they are potentially reasonable options, neither of the remaining answer choices is the best answer because ovarian cancer is the most common form of malignancy in adult white women with dermatomyositis.
127- Which of the following is TRUE?
A. Oral cephalexin should provide significant improvement in this lesion.
B. Excision is the most definitive treatment for this.
C. This patient likely has an underlying immunodeficiency.
D. This patient likely has an underlying psychiatric disorder.
E. Histologically, numerous neutrophils are seen at the edge of the ulcer.
Correct choice: D. This patient likely has an underlying psychiatric disorder.
Explanation: The correct answer is D- this is a factitial ulcer, which is evidenced by the irregular, sharp, geometric borders, lack of surrounding inflammation, and prior sites of picking (with secondary scarring and incomplete alopecia).
128- Which feature of porphyria cutanea tarda is not commonly seen in drug-induced pseudoporphyria?
A. Varioliform scarring
B. Bullous skin reaction
C. Photosensitivity
D. Pain
E. Hypertrichosis
Correct choice: E. Hypertrichosis
Explanation: Hypertrichosis is rarely reported in drug-induced pseudoporphyria.
129- What is the treatment of choice for delusions of parasitosis?
A. Risperidone
B. Doxepin
C. Sertaline
D. Gabapentin
E. Diphenhydramine
Correct choice: A. Risperidone
Explanation: Of all the answer choices, the second generation antipsychotic Risperidone is the best for delusions of parasitosis. The rest of the answer choices can be used to treat itchy and neurogenic pruritus in a number of conditions.
130- Which of the following is characteristic of this diagnosis?
A. This is a reaction from a tick bite.
B. This patient likely has rheumatic fever.
C. A topical preparation that inhibits 14 alpha-demethylase will likely clear this.
D. HIstologically, many plasma cells may be seen.
E. Histologically, parakeratosis with superficial and deep perivascular “cuffing” of lymphocytes may be seen.
Correct choice: E. Histologically, parakeratosis with superficial and deep perivascular “cuffing” of lymphocytes may be seen.
Explanation: The correct answer is E and describes erythema annulare centrifugum, with the characteristic “trailing scale” seen clinically. Erythema migrans does not have scale (A) and this is not the typical appearance for erythema marginatum (B), although both are annular. An azole antifungal will not help this (C) and plasma cells are not characteristic (E).
131- What is the most frequently affected organ in graft-versus-host disease?
A. Stomach
B. Liver
C. Small intestines
D. Esophagus
E. Skin
correct choice: E. Skin
Explanation: Skin is the most affected organ in both acute and chronic GVHD.
132- Which of the following is true regarding this diagnosis?
A. Histologically, many nerve fibers are seen.
B. There is likely polarizable material histologically.
C. This patient was likely born with this.
D. Histologically, there is epidermal hyperplasia and elastic fibers may be decreased.
E. This is often painful.
Correct choice: D. Histologically, there is epidermal hyperplasia and elastic fibers may be decreased.
Explanation: D is the correct answer. This is an acral fibrokeratoma. A and C describe a supernumerary digit, which is usually on the ulnar side of the 5th finger
133- Which of the following is the most appropriate initial treatment?
A. Topical permethrin
B. Oral ivermectin
C. Topical clobetasol
D. Topical tacrolimus
E. Oral methotrexate
Correct choice: C. Topical clobetasol
Explanation: The correct answer is C. Topical steroids are typically first line treatment for lichen planus especially if not widespread.
134- Triggering factors in this generalized variant of psoriasis include all of the following except:
A. pregnancy
B. rapid tapering of sytemic therapies
C. hypocalcemia
D. infections
E. topical irritants
Correct choice: E. topical irritants
Explanation: In generalized pustular psoriasis, the infiltration of neutrophils dominates the histologic picture, while erythema and the appearance of sterile pustules dominate the clinical picture. It is an unusual manifestation of psoriasis, and triggering factors include pregnancy, rapid tapering of corticosteroids (or other systemic therapies), hypocalcemia, infections, and, in the case of the localized pattern, topical irritants. Recently, biallelic mutations in the gene encoding the IL-36 receptor antagonist (leading to increased production of IL-8 and enhanced responses to IL-1β) have been identified in a subset of patients with generalized pustular psoriasis. Generalized pustular psoriasis during pregnancy is also referred to as impetigo herpetiformis.
135- These areas were orange/brown macules that became more raised after examining and manipulating them, as depicted in the photograph. Which of the following stains is LEAST likely to help diagnose this condition histologically?
A. Toluidine blue
B. Giemsa
C. Tryptase
D. CD117
E. SMA
Correct choice: E. SMA
Explanation: The correct answer is E. The picture shows a form of cutaneous mastocytosis and the remainder of the choices will help in identifying mast cells histologically. SMA does not stain mast cells, but a smooth muscle hamartoma is in the differential of a positive Darier sign.
136- A 30-year-old female presents with hypertrophy and nodular enlargement of both lips and pinpoint red mucosal macules. A mucosal biopsy of the lower lip shows inflammatory hyperplasia of the labial salivary glands and dilated secretory ducts that appear inflamed. Which of the following is the most likely diagnosis?
A. Cheilitis granulomatosa
B. Cheilitis glandularis
C. Melkersson-Rosenthal syndrome
D. Actinic cheilitis
E. Pyostomatitis vegetans
Correct choice: B. Cheilitis glandularis
Explanation: Cheilitis glandularis is demonstrated by inflammatory hyperplasia of the lower labial salivary glands that may result from chronic irritation and can arise in association with actinic cheilitis. The secretory ducts of the salivary glands become dilated and inflamed and appear as pinpoint red mucosal macules. The clinical presentation varies from slight hypertrophy of the lower lip to nodular enlargement resulting in eversion. 1 – Cheilitis granulomatosa is a rare, persistent, painless idiopathic chronic swelling of the lip. It is considered a manifestation of orofacial granulomatosis – a term describing orofacial swelling caused by non -caseating granulomatous inflammation in the absence of systemic dise ase. 3 –Melkersson-Rosenthal syndrome is a rare neurological disorder characterized by recurrent facial paralysis, swelling of the face and lips and a fissured tongue. 4 – Actinic cheilitis describes the characteristic changes that occur on the lower vermilion lip of individuals with moderate to severe photodamage. 5 – Pyostomatitis vegetans is seen in patients with inflammatory bowel disease. It is a chronic vegetate pyoderma of the labial and buccal mucosa. It may be associated with vegetative or ulcerat ive pyoderma gangrenosum and is characterized by large verrucous plaques that may be studded with pustules.
137- Which of the following are potential cutaneous manifestations of voriconazole therapy?
A. Cheilitis
B. Pseudoporphyria
C. Eruptive lentigines
D. Squamous cell carcinomas
E. All of the above
Correct choice: E. All of the above
Explanation: These are all potential photosensitive reactions to voriconazole. These resolve on discontinuation of voriconazole. Close
138- A 35-year-old man presents with several round to oval, hypomelanotic macules several centimeters in diameter on his back. There is associated fine scale apparent upon scratching the skin. The distribution is symmetric and is mostly on the posterior trunk and shoulders. While the contrast between the hypomelanotic infected skin and the surrounding normal skin is striking in darkly pigmented patients, or following a tan in lightly pigmented patients, Wood’s lamp
examination may be required to locate all the lesions. Which of the following mechanisms does NOT play a role leading to decreased melanin synthesis in this condition?
A. Azelaic acid
B. Abnormal melanosome production
C. Decreased melanin synthesis
D. Partial block in melanosome transfer to keratinocytes
E. Destruction of melanocytes
Correct choice: E. Destruction of melanocytes Explanation:
In tinea versicolor, the hypopigmented skin, there is a decreased density of melanosomes within keratinocytes, but no change in the melanocyte density. Abnormal melanosome production, decreased melanin synthesis, and a partial block in melanosome transfer to keratinocytes have all been suggested as underlying defects. Azelaic acid, a competitive inhibitor of tyrosinase produced by Malassezia furfur, may play a role in the decreased melanin synthesis. Although the differential diagnosis may include postinflammatory hypopigmentation (e.g. secondary to parapsoriasis), progressive macular hypomelanosis, and early vitiligo, the diagnosis of tinea versicolor is easily confirmed by examination of the associated scale in a potassium hydroxide preparation.
1- Azelaic acid, a competitive inhibitor of tyrosinase produced by Malassezia furfur, may play a role in the decreased melanin synthesis in tinea versicolor.
2- In tinea versicolor, the hypopigmented skin, there is a decreased density of melanosomes within keratinocytes, but no change in the melanocyte density. Abnormal melanosome production, decreased melanin synthesis, and a partial block in melanosome transfer to keratinocytes have all been suggested as underlying defects.
3- In tinea versicolor, the hypopigmented skin, there is a decreased density of melanosomes within keratinocytes, but no change in the melanocyte density. Abnormal melanosome production,
decreased melanin synthesis, and a partial block in melanosome transfer to keratinocytes have all been suggested as underlying defects.
4- In tinea versicolor, the hypopigmented skin, there is a decreased density of melanosomes within keratinocytes, but no change in the melanocyte density. Abnormal melanosome production, decreased melanin synthesis, and a partial block in melanosome transfer to keratinocytes have all been suggested as underlying defects. 5-Vitiligo is a common depigmentation disorder resulting from the destruction of functional melanocytes in the affected skin. In patients with vitiligo, melanocytes are destroyed in the skin, the eyes, and possibly the ears.
139- A 20-year-old female presents with prickling, tingling, burning and a stinging sensation within 30 minutes of water contact, which lasts for up to 2 hours. The symptoms begin on the lower extremities and then generalize, with sparing of the head, palms, soles and mucosa. She notes that the symptoms occur irrespective of the water temperature or salinity. On examination, specific skin lesions are not seen. What is the most likely diagnosis?
A. Polycythemia vera
B. Mastocytosis
C. Hypereosinophilic syndrome
D. Aquagenic pruritus
E. Aquagenic urticaria
Correct choice: D. Aquagenic pruritus
Explanation: Aquagenic pruritus is usually secondary to a systemic disease or another skin disorder such as urticaria or dermatographism. Primary idiopathic aquagenic pruritus is uncommon.
Aquagenic pruritus presents with prickling, tingling, burning, or stinging sensations
within 30 minutes of water contact, irrespective of its temperature or salinity, and lasts for up to 2 hours. Typically, symptoms begin on the lower extremities and then generalize, with sparing of the head, palms, soles, and mucosae. On exam, specific skin lesions are not seen. The pathologic mechanism is unknown although elevated dermal and epidermal level sof acetylcholine, histamine, serotonin, and prostaglandin E2 have been described.
1 - Polycythemia vera: Aquagenic pruritus can sometimes be secondary to a systemic disease such as polycythemia vera. However, the clinical scenario is a classic description of aquagenic pruritus, not polycythemia vera which presents with a ruddy complexion.
2 - Mastocytosis: Mastocytosis usually presents with cutaneous lesions and a positive Darier’s sign. In patients with aquagenic pruritus, specific skin lesions are not seen.
3 - Hypereosinophilic syndrome: Cutaneous lesions are seen in > 50% of patients with hypereosinophilic syndrome. Hypereosinophilic syndrome is on the differential diagnosis of pruritus or prickling sensation provoked by water contact. In patients with aquagenic pruritus, specific skin lesions are not seen.
5 - Aquagenic urticaria: Aquagenic pruritus can sometimes be secondary to another skin disorder such as urticaria or dermatographism. However, the clinical scenario is a classic description of aquagenic pruritus, not aquagenic urticaria. In aquagenic pruritus, specific skin lesions are not seen.
140- You have a patient on both methotrexate and adalimumab and want to avoid live vaccines while they are immunosuppressed. Which of the following vaccines is NOT a live vaccine?
A. Oral polio vaccine
B. MMR (measles, mumps, rubella)
C. BCG
D. Rabies
E. VZV
Correct choice: D. Rabies
141- What is the best treatment for this condition?
A. Dermabrasion
B. Aggressive chemical peels
C. Photoprotection
D. Tanning beds
E. Hydroquinone
Correct choice: C. Photoprotection
Explanation: This is melasma and the best treatment option is aggressive photoprotection with hats and sunscreen. Hydroquinone may work if it is only epidermal melasma. Melasma is made worse by any irritation of the skin therefore aggressive procedures and treatments are typically not recommended.
142- Which drug is a frequent cause of drug-induced pemphigus?
A. Gold
B. Penicillamine
C. TMP-SMX
D. Penicillin
E. Vancomycin
Correct choice: B. Penicillamine
Explanation: Penicillamine is a frequent cause of drug-induced pemphigus.
143- This patient presents with the following rash. What is the most likely diagnosis?
A. Contact dermatitis
B. Scleroderma
C. Eczema
D. Seborrheic dermatitis
E. Dermatomyositis
Correct choice: E. Dermatomyositis
Explanation: This patient has the classic heliotrope rash with shawl sign (photosensitive rash) on the neck/chest, which is most consistent with dermatomyositis.
There is no vesiculation, scale, erosions, lichenification or other features to indicate classic contact dermatitis, seborrheic dermatitis or eczematous dermatitis. The rash in the photo is photo- distributed and suspicious for a connective tissue disease. Scleroderma would have skin tightening rather than erythematous patches.
144- Which of the following is FALSE regarding this diagnosis?
A. Most patients have a preceding history of the plaque-subtype of this diagnosis.
B. Histologically, numerous neutrophils are seen in the epidermis.
C. Acitretin is an effective therapy.
D. This may progress to erythroderma.
E. This may cause loss of the nails if it progresses to the distal digits.
Correct choice: A. Most patients have a preceding history of the plaque-subtype of this diagnosis.
Explanation: The correct answer is A (A is false); patients with pustular psoriasis uncommonly have a preceding diagnosis of plaque type psoriasis
145- Cheilitis granulomatosa is the rare triad of:
A. Large puffy lips, vagus nerve palsy, lingua plicata
B. Lip swelling, beads of mucus on the surface, tenderness
C. Lip swelling, eyelid swelling, thyroid enlargement
D. Lip swelling, lacrimal gland inflammation, lingua plicata
E. Large puffy lips, facial nerve palsy, fissured tongue
Correct choice: E. Large puffy lips, facial nerve palsy, fissured tongue
Explanation: granulomatous cheilitis has been reported with facial nerve palsy and fissured tongue, resulting in the condition known as Melkersson–Rosenthal syndrome.
146- Which of the following are considered risk factors for the development of graft-versus-host disease ?
A. Older age at time of transplant
B. Peripheral blood source
C. HLA incompatible donor
D. Donor is not related to the recipient
E. All of the above
Correct choice: E. All of the above
Explanation: All these are factors related to increased risks of developing graft-versus-host disease.
147- What is the most common ocular finding in this disease with naked granulomas?
A. Iritis
B. Posterior uveitis
C. Cataracts
D. Acute anterior uveitis
E. Blindness
Correct choice: D. Acute anterior uveitis
Explanation: This is sarcoid which histologically shows naked granulomas. The most common ocular finding is acute anterior uveitis. The skin lesions are only found in 25% of patients.
148- Which anticonvulsant and which HLA type would you screen for in Asian patients for higher risk of this potential complication?
A. Carbamazepine; HLA-B1502
B. Carbamazepine; HLA-B51
C. Phenytoin; HLA-B17
D. Phenytoin; HLA-DR4
E. Lamotrigine; HLA-DQw2
Correct choice: A. Carbamazepine; HLA-B1502
Explanation: The correct answer is A. Han Chinese, Malaysian, and Thai populations are at a significantly higher risk of carrying the HLA-B1502 allele and developing SJS/TEN from carbamazepine (but not Japanese or Caucasians).
149- A patient presents with this rash and joint pains after starting a new medication. Which of the following is the most likely?
A. Cefaclor
B. Cefotaxime
C. Cefepime
D. Ceftriaxone
E. Tobramycin
Correct choice: A. Cefaclor
Explanation: Serum sickness-like reactions are most likely secondary to cefaclor.
150- The first-line treatment for the disorder depicted in this image is:
A. high-potency steroids
B. mid-potency steroids
C. low-potency steroids
D. corrective surgery
E. topical calcineurin inhibitors
Correct choice: A. high-potency steroids
Explanation: The main aim of therapy for lichen sclerosus is to bring the disease under control as quickly as possible with the fewest side effects. For initial treatment (after the diagnosis is established), a potent topical corticosteroid such as clobetasol propionate 0.05% ointment is prescribed. One possible regimen is daily application of clobetasol propionate 0.05% ointment to the affected area for 3 months. This alleviates symptoms in the majority of patients, often within weeks. Subsequently, the corticosteroid is tapered down over 2 weeks and the clinical remission maintained by occasional application of a corticosteroid, using less than 30 g in any 6-month period. In some patients there is complete resolution of signs as well as symptoms, with the exception of the scarring, which is irreversible
151- A 55 year old male with medical history significant for type 2 diabetes mellitus, hypertension, and 25 pack/year smoking history presents to the emergency department complaining of sudden onset shortness of breath and hemoptysis. CT scan of the chest/abdomen/pelvis reveals large bilateral pulmonary artery emboli as well as a portal vein thrombus. He is admitted to the ICU where his exam is significant for the findings seen in the image. Lab evaluation reveals eosinophilia and acute kidney injury. What is the most likely diagnosis?
A. Type II Cryoglobulinemia
B. Cholesterol Emboli
C. Warfarin necrosis
D. Henoch Schonlein Purpura
E. Levamisole vasculopathy
Correct choice: B. Cholesterol Emboli
Explanation: The image shows retiform purpura on the the distal lower extremity. Given the history of pulmonary arterial emboli along with eosinophilia and AKI, this is most consistent with cholesterol emboli (choice 2). Although the classic description of this is in a patient who underwent a catheterization, this entity can present from fragmentation of an thrombus. The showers of emboli result in cholesterol deposition in the distal extremities. Type II cryoglobulinemia is a vasculitis, not a vasculopathy, and would present with palpable purpura as opposed to retiform purpura (Type I cryoglobulinemia is a true vasculopathy). The patient has no history to suggest warfarin necrosis, and this entity is similar to calciphylacxis in that it affects more central, fatty areas of the body. Henoch Schonlein Purpura generally affects children (or adults with ESRD), and is a vasculitis which manifests with palpable purpura. There is no history of cocaine use to suggest levamisole vasculopathy.
152- A 32-year-old female presents with a firm, minimally elevated papule that measures 5 mm in diameter and is hyper pigmented. On palpation, the lesion seems attached to subcutaneous tissue. On dermoscopy, a central white scar-like patch surrounded by a delicate pigment network is observed. Which of the following is not a common cause of multiple dermatofibromas?
A. HIV infection
B. Systemic lupus erythematosus
C. Arthropod bites
D. Atopic dermatitis
E. Melanocytic nevi
Correct choice: E. Melanocytic nevi
Explanation: Melanocytic nevi is not a common cause of multiple dermatofibromas. Although some dermatofibromas are thought to arise at sites of trauma or arthropod bites, their precise etiology is not known. Multiple eruptive dermatofibromas have been observed in patients with autoimmune disorders such as lupus erythematosus and atopic dermatitis and in the setting of immunosuppression (i.e. HIV infection). Clinically, dermatofibromas can be confused with cysts or melanocytic nevi, especially those with fibrosis. However, melanocytic nevi are not a common cause of multiple dermatofibromas.
1 - HIV infection: Multiple eruptive dermatofibromas have been observed in patients with autoimmune disorders such as lupus erythematosus and atopic dermatitis and in the setting of immunosuppression (i.e. HIV infection).
2 - Systemic lupus erythematosus: Multiple eruptive dermatofibromas have been observed in patients with autoimmune disorders such as lupus erythematosus and atopic dermatitis and in the setting of immunosuppression (i.e. HIV infection).
3 - Arthropod bites: Some dermatofibromas are thought to arise at sites of trauma or arthropod bites.
4 - Atopic dermatitis: Multiple eruptive dermatofibromas have been observed in patients with autoimmune disorders such as lupus erythematosus and atopic dermatitis and in the setting of immunosuppression (i.e. HIV infection).
1563- What would this look like clinically?
A. Violaceous polygonal papules
B. Skin-colored firm fixed dermal nodule
C. Thick hyperkeratotic plaques
D. Scattered small hyperkeratotic papules centered around hair follicles
E. Juicy papules on the fingers
Correct choice: A. Violaceous polygonal papules
Explanation: Clinically, these lesions are polygonal papules which are consistent with lichen planus. On histology, as seen in the photos, lichen planus is characterized by saw-tooth rete ridges and a lichenoid infiltrate.
154- The following eruption has been associated with which virus(es)?
A. HSV 1 and 2
B. CMV
C. HHV 6 and 7
D. EBV
E. Coxsackievirus A6
Correct choice: C. HHV 6 and 7
Explanation: Pityriasis rosea has been associated with reactivation of HHV 6 and 7.
155- A patient presents with persistent centrofacial erythema, flushing, telangiectasias and skin sensitivity. You suspect he has erythematotelangiectatic rosacea. Which of the following skincare recommendations is appropriate to advise this patient for his skin condition?
A. Procedures such as glycolic peels and dermabrasion
B. Use of waterproof cosmetics
C. Use of cosmetics with alcohol, menthols, camphor, witch hazel, peppermint, and eucalyptus oil
D. Use of astringents, toners, and exfoliators
E. Moisturizers containing humectants and occlusives
Correct choice: E. Moisturizers containing humectants and occlusives
Explanation: Moisturizers containing humectants (e.g. glycerin) and occlusives (e.g. petrolatum) help to repair the epidermal barrier in rosacea. Skincare recommendations for patients with rosacea include:
o Wash with lukewarm water and use soap-free cleansers that are pH balanced
o Cleansers are applied gently with fingertips
o Use sunscreens with both UVA and UVB protection and an SPF ≥30
o Sunscreens containing the inorganic filters titanium dioxide and/or zinc oxide are usually well tolerated
o Use cosmetics and sunscreens that contain protective silicones
o Water-soluble facial powder containing inert green pigment helps to neutralize the perception of erythema
o Moisturizers containing humectants (e.g. glycerin) and occlusives (e.g. petrolatum) help to repair the epidermal barrier
o Avoid astringents, toners, and abrasive exfoliators
o Avoid cosmetics that contain alcohol, menthols, camphor, witch hazel, fragrance, peppermint, and eucalyptus oil
o Avoid waterproof cosmetics and heavy foundations that are difficult to remove without irritating solvents or physical scrubbing
o Avoid procedures such as glycolic peels or dermabrasion
156- Additional manifestations of which disorder range from poikiloderma of the scalp to central flagellate erythema to erosions and ulcerations:
A. systemic lupus erythematosus
B. scleroderma
C. sarcoidosis
D. dermatomyositis
E. Behcet’s disease
Correct choice: D. dermatomyositis
Explanation: Additional manifestations of dermatomyositis range from poikiloderma of the scalp to centripetal flagellate erythema to erosions and ulcerations. It is also important to look for dermatologic signs of other connective tissue diseases in patients with dermatomyositis, because of the frequency of overlap syndromes. The most common cutaneous findings would be signs of overlap with systemic sclerosis, especially limited disease with CREST features (calcinosis, Raynaud’s phenomenon, esophageal dysmotility, sclerodactyly, telangiectasias), annular lesions suggestive of subacute cutaneous lupus erythematosus, or nodules suggestive of rheumatoid arthritis. Cutaneous small vessel vasculitis can occur as an associated finding, especially in patients with juvenile dermatomyositis.
157- A patient has multiple erythematous/violaceous psoriasiform dermatitis affecting the ears, nose, hands and feet. The patient is diagnosed with Bazex syndrome. The most appropriate course of action is to:
A. To do an EGD
B. Use topical corticosteroids
C. To do a CT scan of the abdomen
D. To order lipase and amylase levels
E. To do a CT of the chest
Correct choice: A. To do an EGD
Explanation: A patient with Bazex syndrome also known as Acrokeratosis Paraneoplastica should have an EGD. Patients can also have nail dystrophy , acquired keratoderma, and the skin findings usually precede the underlying malignancy. The other listed testing is not required for Bazex syndrome.
158- This 17-year-old boy also reports dark urine, bilateral knee pain, and intermittent abdominal pain. Which of the following is true regarding this condition?
A. He has a high likelihood of developing permanent renal impairment
B. Azathioprine is the treatment of choice
C. It is more common in adults than children
D. It usually occurs after a respiratory tract infectioon
E. A DIF biopsy showing intravascular IgA deposition is diagnostic
Correct choice: D. It usually occurs after a respiratory tract infectioon
Explanation: This patient has IgA vasculitis, also known as Henoch-Schonlein purpura (HSP). HSP typically occurs after a respiratory tract infection in children.
HSP is much less common in adults. A DIF biopsy showing perivascular IgA deposition is diagnostic (C3 and fibrin may also deposit perivascularly). Two percent of patients develop permanent renal impairment. Treatment is mainly supportive as HSP is typically self-limited. Corticosteroids may be effective in rapid resolution of renal manifestations and treating joint and abdominal pain, but they are not proven effective for treating organ manifestations and complications, such as glomerulonephritis, bowel infarction or intussusception. Mycophenolate mofetil or cyclosporine A may be better treatment choices in case of renal involvement. Other immunosuppressive and immunomodulating drugs, such as rituximab and dapsone, are promising, but larger studies are needed to confirm these findings. Cancer screening should be considered in older males diagnosed with Henoch-Schönlein purpura.
159- This 36-year-old woman presents to the ED for recent-onset progressive shortness of breath. She is otherwise healthy and takes no medications. She denies fever, chills, dysphagia, chest pain, nausea, vomiting, joint pain or muscle weakness. You are called to evaluate the pictured skin findings. Which of the following autoantibodies is most likely positive in this woman?
A. MDA5
B. TIF1-gamma
C. Jo-1
D. SRP
E. Mi-2
Correct choice: A. MDA5
Explanation: This patient’s findings of ragged cuticles (Samitz sign), periungual erythema, and dorsal finger erosions are highly suggestive of the subtype of dermatomyositis (DM) with positive MDA5 (melanoma differentiation-associated gene 5) autoantibodies. This subtype is usually amyopathic or hypomyopathic, explaining this patient’s lack of muscle weakness. It is also associated with rapidly-progressive interstitial lung disease, which accounts for her pulmonary symptoms.
Mi-2 autoantibodies are associated with classic manifestations of DM (heliotrope rash, Gottron papules) and a positive prognosis. SRP autoantibodies are associated with DM with cardiac involvement and poor prognosis. TIF1-gamma autoantibodies are associated with paraneoplastic DM. Lastly, the DM anti-synthetase syndrome often has Jo-1 autoantibodies and is characterized by pulmonary disease, arthritis, Raynaud’s, and myositis.
160- Which of the following is FDA-approved to treat oral ulcers associated with Behcet’s syndrome?
A. Hydroxychloroquine
B. Mycophenolate mofetil
C. Apremilast
D. Azathioprine
E. Methotrexate
Correct choice: C. Apremilast
Explanation: The small-molecule phosphodiesterase 4 inhibitor apremilast modulates cytokines that are up-regulated in Behçet’s syndrome. In patients with oral ulcers associated with Behçet’s syndrome, apremilast resulted in a greater reduction in the number of oral ulcers than placebo but was associated with adverse events, including diarrhea, nausea, and headache. Consequently, in July 2019, apremilast was FDA-approved for the treatment of oral ulcers associated with Behcet’s syndrome. The remaining answers are not FDA-approved to treat oral ulcers associated with Behcet’s syndrome.
161- What is the treatment of choice?
A. Isotretinoin
B. Insulin
C. Minocycline
D. Topical corticosteroids
E. Selenium sulfide
Correct choice: C. Minocycline
Explanation: The picture shows confluent and reticulated papillomatosis. There are coalescing and hyperkeratotic papules located on the central trunk. Minocycline is the most effective treatment; however, recurrence is common.Insulin may be needed for patients with acanthosis nigricans, a cutaneous marker of insulin resistance and metabolic disorders. Selenium suflide is the treatment of choice for tinea veriscolor. Tinea veriscolor is characterized by hypo or hyperpigmented macules and papules with fine scale. The diagnosis can be confirmed with a potassium hydroxide prep and visualization of clustered yeast and hyphae.Topical corticosteroids are ineffective for the treatment of confluent and reticulated papillomatosis. Isotretinoin has been reported to be effective in the treatment of confluent and reticulated papillomatosis; however, in light of the high likelihood of recurrence and side effect profile, it is not a first line treatment.
162- This 34-year-old woman reports a long history of Raynaud’s phenomenon. The only medication she takes is albuterol as needed for exercise-induced asthma. On exam, you notice splinter hemorrhages of the fingernails as well as the pictured findings. Which of the following antibodies is LEAST likely to be positive in this woman?
A. Anti-nuclear
B. Anti-B2 glycoprotein
C. Anti-cardiolipin
D. Lupus anticoagulant
E. Anti-histone
Correct choice: E. Anti-histone
Explanation: This patient is presenting with signs and symptoms concerning for anti-phospholipid antibody syndrome (APS). Anti-histone antibodies are typically only present in the setting of drug- induced systemic lupus erythematosus (SLE), which is extremely unlikely given that this patient’s only medication is albuterol. Common culprit medications for drug-induced SLE are procainamide, hydralazine, isoniazid, and minocycline. Patients with APS commonly have the remaining listed positive antibodies. APS is an autoimmune disease characterized by antiphospholipid antibodies (aPL) associated with thrombosis and/or pregnancy morbidity. Most APS events are directly related to thrombotic events, which may affect small, medium or large vessels. Other clinical features like thrombocytopenia, nephropathy, cardiac valve disease, cognitive dysfunction and skin ulcers (called non-criteria manifestations) add significant morbidity to this syndrome and represent clinical situations that are challenging. APS was initially described in patients with systemic lupus
erythematosus (SLE) but it can occur in patients without any other autoimmune disease. Despite the autoimmune nature of this syndrome, APS treatment is still based on anticoagulation and antiplatelet therapy.
163- A plain film showing “net-like calcification” may obviate the need for biopsy, potentially significantly increasing mortality if secondary ulceration occurs, in which of the following conditions?
A. Pancreatic panniculitis
B. Scleroderma
C. Wegener’s granulomatosis
D. Calciphylaxis
E. Pseudoxanthoma elasticum
Correct choice: D. Calciphylaxis
Explanation: The correct answer is E. Xrays of the affected limb may demonstrate vascular calcification within the skin; however, this may also be seen in healthy patients with renal disease that are not affected by calciphylaxis.
Bone scintigraphy using technetium Tc 99m bisphosphonates in patients with calciphylaxis shows increased radiotracer uptake in soft tissues throughout the body and is specifically enhanced in indurated plaques affected by calciphylaxis (but is absent in ulcers due to reduced blood flow at sites of tissue necrosis). Calciphylaxis is a condition characterised by necrosis (cellular death) of the skin and fatty tissue. It is seen mainly in patients with end-stage kidney disease. It is also sometimes called calcific uraemic arteriolopathy or calcific vasculopathy.
In 1981, approximately 50 cases of calciphylaxis were reported in the world literature. Today, the incidence is estimated at 1 per cent per year in patients undergoing renal dialysis. The mortality is extremely high, up to 80%, often within several months of onset. The primary cause of death is from secondary infection of the ulcers, and sepsis. Calciphylaxis is also called calcific uraemic arteriolopathy or calcific vasculopathy. A deep wedge skin biopsy may be necessary to diagnose calciphylaxis, as a similar appearance can be seen in other conditions such as necrotizing fasciitis, Cryoglobulinemia, antiphospholipid syndrome, coumarin necrosis, and vasculitis. Multiple biopsies may be necessary, with a risk of propagating calciphylaxis. The pathologist looks for calcium deposited within scarred and blocked blood vessels in the subcutaneous tissue. Perieccrine calcium deposition may be noted when vascular calcification is absent but may be subtle. There may also be inflammation of the fat (panniculitis).
164- Which of the following is FALSE?
A. This typically occurs on the extremities, especially legs.
B. Histologically, a parakeratotic column with underlying dyskeratotic keratinocytes may be seen.
C. The plaque (Mibelli) subtype of this diagnosis typically appears in childhood.
D. Treatment of these lesions is usually unsuccessful.
E. The palmoplantar subtype of this diagnosis has the greatest malignant potential of all subtypes.
Correct choice: E. The palmoplantar subtype of this diagnosis has the greatest malignant potential of all subtypes.
Explanation: The correct answer is A (A is false). Linear porokeratosis has the greatest risk of malignant (SCC) transformation of all subtypes of porokeratosis. This photo depicted is disseminated superficial actinic porokeratosis (DSAP) which is seen on the legs>arms and is very difficult to treat. E describes the cornoid lamella characteristically seen in porokeratosis. The plaque (Mibelli) subtype of this diagnosis typically appears in childhood. The palmoplantar subtype of this diagnosis does not have the greatest malignant potential of all subtypes.
165- What is the treatment of choice?
A. Incision and drainage
B. Excision
C. Topical antibiotics
D. Tooth extraction
E. Oral antibiotics
Correct choice: D. Tooth extraction
Explanation: The image is of a dental sinus tract, an intraoral connection with a periapical abscess. Extraction of the nonviable tooth is essential to treatment. Topical antibiotics are not an effective treatment for dental abscesses. Oral antibitoics and incision and drainage may be needed in addition to tooth extraction for infection eradication. An excision would not address the primary problem, which is a chronic dental infection.
166- A patient presents with a rash on his back for a 4-week duration. History elicits a new couch purchase prior to the onset of this rash. Which of the following allergens is the most likely culprit?
A. Colophony
B. Chromate
C. Dimethyl fumarate
D. Quaternium-15
E. Thiuram
Correct choice: C. Dimethyl fumarate
Explanation: Dimethyl fumarate, the contact allergen of 2011, is present in sachets to prevent the mold formation on shipped furniture. Thiuram is typically present in rubber, latex, and adhesives.
Colophony or rosin is found in adhesive tape, cosmetics, insulating tape, glossy paper, polish, paints, inks, epilation wax, varnishes, violin bows, and chewing gum. Chromate is found in leather, cements, and some green felts (pool tables). Quaternium-15 is the most common cosmetic preservative to cause ACD. It is a preservative that releases formaldehyde.
167- Which of the following is LEAST likely to produce the pictured skin finding?
A. Pityriasis rubra pilaris
B. Mycosis fungoides
C. Psoriasis
D. Seborrheic dermatitis
E. Atopic dermatitis
Correct choice: D. Seborrheic dermatitis
Explanation: This man has erythroderma. Seborrheic dermatitis is a rare cause of erythroderma in young children, and would be an extremely unlikely cause in an adult. Pre-existing dermatoses (primarily atopic dermatitis or psoriasis), medications, cutaneous T-cell lymphoma (especially the leukemic form of mycosis fungoides called Sezary syndrome), and pityriasis rubra pilaris are more common causes of erythroderma than seborrheic dermatitis in adults.
168- Which of the following is a feature of Schnitzler’s Syndrome?
A. Monoclonal IgG gammopathy
B. Thromboembolic events
C. Bronchospasm
D. Hematuria
E. Sensorimotor neuropathy
►E
Schnitzler’s syndrome presents as episodes of urticarial vasculitis that occur in association with a monoclonal IgM M component. Fever, lymphadenopathy, hepatosplenomegaly, bone pain, and sensorimotor neuropathy also occur.
169 -TH2 immune responses:
A. Are associated with cell-mediated immunity
B. Produce IL-6
C. Produce IFN-gamma
D. Produce TNF-beta
E. Produce IL-2
►B
TH1 cells produce IL-2, IFN-gamma, and TNF-beta, and are associated with cell-mediated immunity. TH2 cells produce IL-4, IL-5, IL-6, IL-10, and IL-13, and are associated with antibody- mediated immune responses.
170 -What is the treatment of choice for this condition?
A. Nystatin
B. Fluconazole
C. Better oral hygiene
D. Penicillin
E. Acyclovir
►C
Black tongue is associated with poor oral hygiene, the use of medications, and radiation to the head and neck region. In many cases, simply brushing the tongue with a toothbrush or using a commercially available tongue scraper is sufficient improve the condition.
171- The differential diagnosis for the attached image should include all of the followings except
A. Candidiasis
B. Lichen planus
C. Contact dermatitis
D. Inverse psoriasis
E. Erythrasma
►E
The image shown is inverse type of psoriasis. Although psoriatic lesions are classically distributed on the extensor surfaces (the elbows, knees, and lumbosacral regions), lesions may also be found in a flexural distribution with involvement of the axillae, groin, perineum, central chest, and umbilical region. Differential diagnosis include all other choices in question. Although erythrasma might be included in differentials in case of groin involvement only, it is unlikly for eryhrasma to presnt on penis with well demarctaed erythamtos plaques. Erythrasma is a superficial bacterial infection of the skin caused by C. minutissimum. It is characterized by asymptomatic, well-demarcated, reddish brown, slightly scaly patches in the groin, axillae, gluteal crease, or inframammary regions, and less often the interdigital spaces of the feet. Erythrasma is frequently confused with a dermatophyte infection, it can be differentiated from tinea infection by the characteristic coral red fluorescence seen when viewed under Wood’s lamp illumination (due to the production of porphyrins by the corynebacteria.
172- Which of the following is a progestin used in oral contraceptives with low intrinsic androgenic properties?
A. Norgestrel
B. Novicane
C. Norgestimate
D. Levonorgestrel
E. Spironolactone
►C
Desogestril, norgestimate and gestodone are progestins with low intrinsic androgenic properties.
173 -Patients that have this form of acne is seen affecting young males with suppurating nodules and plaques that ulcerate and form blackish eschar seen on the trunk most of the time. These patients can have leukocytosis, fever, arthralgia and lytic changes in the bone most commonly affecting the:
A. Sternoclavicular joint and chest wall
B. Shoulder blades and sternum
C. Elbows and knees
D. Neck and sternum
E. Jaw and frontal bone
►A
Patients with acne fulminans can have severe acne lesions with lytic lesions indicative of sterile osteomyelitis seen on x-ray and bone scans. The sternoclavicular joint and the chest wall are most frequently affected. Treatment includes oral prednisone, antibiotics and isotretinoin.
174- Efficacy of oral contraceptives is reduced by co-administration with which of the following drugs?
A. Minocycline
B. Doxycycline
C. Rifampin
D. TMP-SMX
E. All of these answers are correct.
►C
Rifampin is a potent hepatic microenzyme inducer and has been shown to reduce efficacy of oral contraceptives. Anecdotal reports of decreased contraceptive efficacy on oral antibiotics have not been confirmed in the literature. The putative mechanism is decreased absorption of hormones secondary to altered gut microflora.
175- Secondary systemic amyloidosis:
A. Classically involves the tongue and periorbital skin
B. Involves deposition of AL protein
C. Can involve deposition of beta 2-microglobulin in the setting of rheumatoid arthritis
D. Can be noted on biopsy of normal skin
E. Involves deposition of keratin-derived amyloid
►D
Secondary systemic amyloidosis presents with deposition of amyloid in the adrenals, liver, spleen, and kidney as a result of some chronic disease, such as TB, leprosy, Hodgkin’s, Behcet’s, rheumatoid arthritis, ulcerative colitis, schistosomiasis, or syphilis. The skin is not involved. AA amyloid fibrils, derived from SAA protein (an acute phase reactant) are deposited. AA is also seen in Muckle-Wells and familial mediterranean fever. Biopsy of normal skin may be positive for perivascular amyloid. Dialysis-related amyloidosis occurs via deposition of beta 2microglobulin component altered by uremia, and resuls in carpal tunnel syndrome, bone cysts, and spondyloarthropathy.
176- A 35 year old man with a history of celiac disease presents with a beefy, red tongue, hyperpigmented palmar creases, and premature grey hair. Which of the following statements are is correct?
A. The best therapy is riboflavin 5mg/day
B. This condition mimics folate deficiency
C. It is often associated with carcinoid tumors which divert tryptophan to serotonin
D. This condition can be caused by azithioprine, 5-FU, and isoniazid
E. Eating raw egg whites is a risk factor
►B
This condition is vitamin B12 deficiency and is characterized by glossitis and hyperpigmentation in sun exposed areas and creases. Neurologic abnormaolities and megaloblastic anemia can be seen. The symptoms can mimic folic acid deficiency. Riboflavin (B2) is associated with oralocular- genital syndrome. Carcinoid tumors as well as azathioprine, 5-FU, and isoniazid are associated with niacin deficiency. Eating raw eggs is a risk factor for biotin deficiency.
177- Which of the following HLA types is associated with early onset psoriasis?
A. HLA-B13 or HLA-B17
B. HLA-B27
C. HLA-B57, HLA-Cw6, or HLA-DR7
D. HLA-Cw2
E. HLA-DR3
►C
HLA-B57, HLA-Cw6, or HLA-DR7 are most commonly associated with early onset type I psoriasis. The presence of HLA-B13 or B17 is associated with a 5-fold risk of developing psoriasis and are increase in guttate and erythrodermic psoriasis. HLA-B27 may be seen in pustular psoriasis. HLA-Cw2 is seen with late onset psoriasis, or type II. HLA-DR3 is commonly found with subacute cutaneous lupus
178- Monotherapy for acne with topical antibiotics is discouraged because of:
A. Slow onset of comedolytic action
B. Potential for irritation
C. Lack of anti-inflammatory action
D. Potential for bacterial resistance
E. Poor patient compliance
►D
Topical antibiotics reduce the population of P. acnes on the skin, and thus are indirectly anti inflammatory. In contrast to topical retinoids, topical antibiotics are not comedolytic. They are generally well tolerated by patients.
179 -What condition does this patient have given the extent of arthritis seen in this X-ray?
A. Antiphospholipid syndrome
B. Multicentric reticulohistiocytosis
C. Alpha-1-antitrypsin syndrome
D. Eosinophilic fasciitis
E. Dermatomyositis
►B
Multicentric reticulohistiocytosis is a non-Langerhans histiocytosis associated with mutilating arthritis. Dermatologically, patient may present with coral beading around the fingers. There is a 30% incidence of malignancy in these patients.
180- A 16 year old developmentally normal male presents to his pediatrician intermittent vague epidodes of hand and feet paresthesias and non specific episode of GI distress. He is referred to you to evaluate numerous punctate to 5 slightly verrucous, deep-red to blue-black papules distributed diffusely on his trunk in a bathing suit distribution. Polarization microscopy of the sediment of his urine demonstrates birefringent lipid globules (ie, renal tubular epithelial cells or cell fragments with lipid inclusions) with the characteristic Maltese cross configuration. How is this disorder inherited?
A. Autosomal dominant
B. Autosomal recessive
C. X-linked dominant
D. X-linked recessive
E. Not an inherited disorder
►D
Fabry’s disease is inherited is x-linked recessive. Female carriers often exhibit mild forms of this disorder. It can be establish that a patient has FD by searching for low activity of alpha galactosyl A in plasma, leukocytes, cultured skin fibroblasts, or dried blood spots on filter paper. Because of the Lyon effect, enzymatic detection of carriers can be misleading; thus, specific genetic analysis can be helpful in making the diagnosis.
181- Mutations in the MEFV gene product, pyrin, produce an autosomal recessive syndrome characterized by recurrent fevers, peritonitis, pleuritis, arthritis and erysipelas-like erythema. Which of the following syndromes is described above?
A. Familial Mediterranean Fever
B. PAPA syndrome
C. TNF receptor associated periodic syndrome
D. Hyper IgD syndrome
E. Familial cold autoinflammatory syndrome
►A
Familial Mediterranean Fever is described above. It is autosomal recessive with incomplete penetrance. The main therapy is Colchicine. PAPA syndrome, TNF receptor associated periodic syndrome, hyper-IgD syndrome and familial cold autoinflammatory syndrome are related autoinflammatory syndromes. PAPA syndrome is Pyogenic Arthritis, Pyoderma gangrenosum and Acne and is caused by proline serine threonine phosphatase-interacting protein (PSTPIP1) or CD2- binding protein 1 (CD2BP1) which interact with pyrin. TNF receptor associated periodic syndrome (TRAPS) has similar findings to FMF, but the attacks are of longer duration, is autosomal dominant in transmission, and does not respond to colchicine. TRAPS is caused by a mutation in the TNFRSF1A gene which results in a decrease in soluble TNF receptor. Hyper IgD syndrome is associated with defects in the mevalonate kinase gene, which presents with a hereditary periodic fever. Familial cold autoinflammatory syndrome is associated with cryopyrin defects and is characterized by fever, rash, conjunctivitis and arthralgia upon exposure to cold.
182- Which of the following is important in the pathogenesis of acne vulgaris?
A. Activation of toll-like receptor-3 by P. acnes
B. Activation of toll-like receptor-2 by M. furfur
C. Activation of toll-like receptor-2 by P. acnes
D. P. acnes produces lipase which cleaves cholesterol into triglycerides
E. Demodex activates complement
►C
Acne vulgaris is a disease of follicular hyperkeratosis and the microcomedone is thought to be the precursor lesion. P. acnes has lipase that cleaves triglycerides into free fatty acids. P. acnes can activate complement and PMN chemotaxis. Toll-like receptors (TLR) recognize bacterial patterns and P. acnes activates TLR-2.
183- A patient is diagnosed with glucagon secreting tumor of the pancreas with painful scaling erythematous patches and plaques at pressure and friction sites. The cells that involved in t his tumor are:
A. alpha islet cells
B. beta islet cells
C. glycon cells
D. insulin cells
E. gamma islet cells
►A
The cells secreting the glucagon tumor are the alpha islet cells of the pancreas. This patient has necrolytic migratory erythema which is associated with pancreatic glucagonoma, malignancies, liver disease, malabsorption states, and inflammatory bowel disease.
184- A patient with holocarboxylase synthetase deficiency is most at risk for developing a deficiency in:
A. Niacin
B. Pyridoxine
C. Biotin
D. Zinc
E. Essential fatty acids
►C
There are three autosomal recessive syndromes that can lead to biotin deficiency: holocarboxylase synthetase deficiency, biotinidase deficiency, and an inability to transport biotin into cells. All of these syndromes present with a dermatitis similar to that of zinc deficiency as well as neurologic abnormalities
185- The most appropriate treatment for the condition shown is:
A. Topical clindamycin 1% lotion
B. Topical metronidazole 0.75% gel
C. Azelaic acid 20% cream
D. Salicylic acid 2% wash
E. Adapalene 0.1% gel
►E
This slide shows comedonal acne. Topical retinoids are the treatment of choice. Salicylic acid containing cleansers may be helpful adjuncts to treatment with mild comedolytic properties.
186 -A patient with this cutanous finding along with peri-ocular erythema, deltoid weakness and pulmonary disease is most likely to have which laboratory finding?
A. Anti-Jo-1 antibodies against histidyl-tRNA synthetase
B. Anti-Jo-1 antibodies against nuclear helicase
C. c-ANCA against proteinase-3
D. Lupus anticoagulant
E. Anti-histone antibodies
►A
Anti-Jo-1 antibodies against histidyl-tRNA synthetase are described in patients with dermatomyositis and pulmonary disease. Anti-Mi-2 antibodies against nuclear helicase are described in classic DM with a good prognosis. Patients with Wegener’s disease display cANCA antibodies against proteinase-3 in a cytoplasmic pattern. Antiphospholipid antibodies are also known as anticardiolipin antibodies and lupus anticoagulant. They can cause a false positive VDRL. Clinical features include livedo reticularis, thrombotic events and spontaneous abortions. Anti- histone antibodies are seen in drug-induced lupus.
187- Natural infection with which of the following infectious agents has been shown to mitigate atopic dermatitis?
A. Measles
B. S. aureus
C. Varicella
D. Rubella
E. Coxsackie virus
►A
Natural measles infection has been shown to ameliorate atopic dermatitis.
188- A 32 year-old pregnant woman presents for treatment comedonal and moderately inflammatory acne. You discuss treatment options with her, and she asks about the evidence for safety in pregnancy of various treatments. According to the FDA classifications, which of the following treatments either shows no risk to the fetus in controlled studies (but may show risk to animals), or shows no risk in animal studies (but no human studies have been conducted)?
A. Benzoyl peroxide
B. Trimethoprim-sulfamethoxazole
C. Topical tretinoin
D. Azeleic acid
E. Tetracycline
►D
The FDA classifies medication safety for use in pregnancy as follows: Category A: Controlled studies in humans show no risk to fetus. Category B: Controlled human studies show no risk (but may show risk to animals), or no risk in animal studies (but no human studies have been conducted). Category C: Risk to human fetus cannot be ruled out, studies are lacking; animal studies are equivocal. Category D: Controlled studies show risk, but in some instances benefits may outweigh risks. Category X: Contraindicated in pregnancy. The question describes category B. Azeleic acid is a category B medication. Benzoyl peroxide, topical tretinoin, and TMP/SMX are category C medications. Tetracycline is a category D medication.
189- Kveim-Siltzback test is useful in the diagnosis of:
A. Scarlet fever
B. Sarcoidosis
C. Leprosy
D. Leishmaniasis
E. Chancroid
►B
Kveim-Siltzback test is for sarcoidosis, Montenegro-leishmanin test is for Leishmaniasis, and lepromin test or histamine/methacholine sweat test can be used for leprosy. Kveim -Siltzback test is done by intradermal injection of a suspension from granuloma-containing spleen, lymph node, or other tissue from a confirmed case of sarcoidosis. A positive test is characterized by the formation of a papule at the site of injection within 4-6 weeks which, on microscopic examination, exhibits non-necrotizing granulomas and the absence of foreign material. This teast is rarely done nowdays because of the difficulties involved in preparation, standardization and validation of the test material as well as significant variation in the sensitivity and specificity of test suspensions obtained from different sources.
190- Patients that have a persistent mongolian spot with indistinct borders that are persistent and or progressive in behavior may have a sign of:
A. GM1 gangliosidosis type I
B. Handlers Disease
C. Mosaic hypopigmentation
D. Neurofibromatosis type II
E. Nevus anemicus
►A
A patient with a persistent mongolian spot with indistinct borders, persistent or progressive behavior may have an underlying lysosomal storage disease, most commonly GM1 gangliosidosis type I and Hurler disease.
191- A patient has multiple erythematous/violaceous psoriasiform dermatitis affecting the ears, nose, hands and feet. The patient is diagnosed with Bazex syndrome. The most appropriate course of action is to:
A. Do an age appropriate cancer screening.
B. Use topical corticosteroids
C. To do a CT scan of the abdomen
D. To order lipase and amylase levels
E. To do a CT of the chest
►A
A patient with Bazex syndrome also known as Acrokeratosis Paraneoplastica should have an age appropriate cancer screening. Patients can also have nail dystrophy , acquired keratoderma, and the skin findings usually precede the underlying malignancy.
192- A patient with inflammatory bowel disease develops acute tender juicy plaques on the head and neck, fever, and malaise. The skin lesions respond well to prednisone. Data shows increased ESR and neutrophilic infiltrate with dermal edema on skin biopsy. Which one of the following is a major criteria for this condition?
A. Fever and malaise
B. History of inflammatory bowel disease
C. Abrupt onset of plaques
D. Increased ESR
E. Good response to prednisone
►C
This condition is called Sweet’s syndrome, or acute febrile neutrophilic dermatosis. Diagnosis relies on two major and two minor criteria. Major ones include 1) an abrupt onset of juicy painful plaques and bullae and 2) neutrophilic infiltration in the dermis on pathology. Minor criteria include 1) presence of associated conditions ie inflammatory bowel disease, infections, pregnancy, leukemia, etc 2) fever and malaise, 3) laboratory values ie high ESR and CRP, and 4) excellent response to prednisone.
193- Which of the following is associated with Reiter‘s syndrome:
A. Ulcerative colitis
B. Multiple sclerosis
C. HLA-Cw6
D. Asymmetric arthritis
E. Perioral dermatitis
►D
Reiter‘s syndrome is a chronic inflammatory disease similar to psoriatic arthritis. Commonly, patients present with either a peripheral, asymmetric arthritis, or a urethritis, or conjunctivitis. Not all of these findings need be present. Patients are usually men with the HLA -B27 genotype. Other clinical findings include fever, weight loss, keratitis, iritis, and cardiac disease. Skin lesions are often found on the palms (keratoderma blenorrhagicum _ hyperkeratotic papules and plaques) or the penis (balanitis circinata). Reiter‘s follows an infection of either the genitourinary tract or the gastrointestinal tract.
194- Which form of sarcoidosis is associated with camptodactyly?
A. Lofgren’s syndrome
B. Heerfordt’s syndrome
C. Darier-Roussy
D. Mikulicz syndrome
E. Blau syndrome
►E
Camptodactyly is a flexion contracture of the 3rd through the 5th proximal interphalangeal joints and elbows bilaterally. Blau syndrome is an autosomal dominant form of sarcodosis due to a defect in the CARD15 gene manifesting also with arthritis, cutaneous sarcoid, uveitis, and synovial cysts. It does not have lung or visceral involvement.
195 -Increased chylomicrons are a feature of which type of hyperlipoproteinemia?
A. Type I
B. Type IIa
C. Type IIb
D. Type III
E. Type IV
►A
Type I (familial lipoprotein lipase deficiency or apoprotein CII deficiency) hyperlipoproteinemia has increased chylomicrons as a feature, and presents with eruptive xanthomas and lipemia retinalis. For the other hyperlipoproteinemias, the associated lipid abnormalities are as follows: Type IIa – increased LDL; Type IIb – increased LDL and VLDL; Type III – increased IDL; Type IV – increased VLDL. Type V has increased chylomicrons and increased VLDL.
196- Which systemic anesthetic can be safely used in mastocytosis?
A. Lidocaine
B. Succinylcholine
C. D-tubocurarine
D. Thiopental
E. Propofol
►E
Local lidocaine can be safely used but systemic lidocaine can precipitate anaphylaxis in patients with mastocytosis. Other systemic analgesics that are contraindicated in mastocytosis include succinylcholine, d-tubocurarine, thiopental, metocurine, enflurane, isoflurane, and etomidate. Propofol, vecuronium bromide, and fentanyl are safe options for systemic anesthesia in patients with mastocytosis.
197- A patient with spontaneous occurrence of petechiae and purpura, particularly around the eyes, will also most likely have which of the following?
A. Diffuse scaling of the scalp
B. Acne
C. Cheilits
D. Macroglossia
E. Uveitis
►D
This patient has primary systemic amyloidosis. Mucocutaneous findings can be seen in up to 40% of patients. The surface of the tongue may be smooth and dry or covered with waxy papules and nodules. Teeth indentations may be seen along the lateral borders. The most common cutaneous signs are petechiae and purpura that occur spontaneous or after minor trauma (“pinch purpura” about the eyelids). The most characteristic skin lesions are waxy, shiny, smooth papules and nodules which are usually hemorrhagic or flesh-colored. Flexural areas are common sites. These patients may also present with carpal tunnel syndrome, hepatomegaly, and edema.
198- A teenage girl presents with recurrent upper eyelid edema and upper vermillion and cutaneous lip swelling, giving her the appearance of having a ―double upper lip.ǁ She reports that the
swelling eventually self-resolves over the course of several days. Additionally, she is noted to have an enlarged, non-tender thyroid gland upon palpation. She is most likely to suffer from which of the following conditions?
A. Ascher syndrome
B. Hereditary angioedema
C. Sarcoidosis
D. Airborne allergic contact dermatitis
E. Granulomatous cheilitis
►A
Ascher syndrome is a rare syndrome characterized by recurrent upper (and sometimes lower) eyelid swelling as well as upper vermillion and cutaneous lip swelling. The latter often yields the appearance of a ―double upper lip,ǁ while multiple episodes of the former results in stretching and redundancy of the eyelid skin, causing blepharochalasis. Ascher syndrome is frequently seen in association with a euthryoid (non-toxic) goiter, although up to 50% of patients may have no associated thyroid abnormality. The pathogenesis of this condition remains unknown.
199- Follicular lichen planus of the skin, multifocal cicatricial alopecia of the scalp and nonscarring alopecia of the axillary and pubic areas area characteristics of:
A. Graham-Little-Piccardi-Laassueur syndrome
B. Erosive lichen planus
C. Lichen planus pigmentosus
D. Vesiculobullous lichen planus
E. Atropic lichen planus
►A
Graham-Little-Piccardi-Lassuer syndrome has a triad of follicular LP of the skin and or scalp, multifocal cicatricial alopecia of the scalp and non-scarring alopecia of axillary and pubic areas.
200 -What organism caused this disease?
A. Micrococcus sedenditaris
B. Candida albicans
C. Staphylococcus aureus
D. Corynebacterium tenuis
E. Corynebacterium diptheria
►A
Pitted keratolysis is a skin disorder characterized by crateriform pitting that primarily affects the pressure-bearing aspects of the plantar surface of the feet. It is caused by a cutaneous infection with micrococcus sedentarius which is included in the Corynebacteria genus. These are grampositive, catalase-positive, aerobic or facultatively anaerobic, generally nonmotile rods. Corynebacterium diphtheriae infection is classically characterized by a local inflammation, usually in the upper respiratory tract, associated with toxin-mediated cardiac and neural disease. Corynebacterium tenuis causes trichomycosis Patients typically present with yellow, black, or red pinpoint nodules on the hair shafts in the inguinal region. Staphylococcus aureus is a common bacterium that can result in formation of pustules, furuncles, cellulitis and abscesses. Candida species are responsible for superficial infections such as oropharyngeal candidiasis (thrush) and vulvovaginal candidiasis (vaginal Candidiasis) and is also occurs as an opportunistic infection
201- A 57yo male with hx of hypothyroidism and SCC of the head and neck s/p XRT several years ago presented with a skin eruption present on his back and upper arms for the past several months. What is the most likely diagnosis?
A. Tinea corporis
B. Granuloma annulare
C. Metastatic squamous cell carcinoma
D. Radiation dermatitis
E. Mycosis fungoides
►B
It is most likely a granulomatous inflammatory disease with infiltrates of macrophages arranged around focus of degenerated collagen and mucin. The etiology is unknown: metabolic disturbances, autoimmune, allergy, and infection.
- A- patient has many painful oral erosions/stomatitis with many lichenoid lesions, erythemamultiforme-like lesions and flaccid bullae. The patient is diagnosed with paraneoplastic pemphigus and is associated with all of the following malignancy except:
A. Non-Hogkins lymphoma
B. Chronic lymphocytic leukemia
C. Thymoma
D. Castleman’s tumor
E. Gastric carcinoma
►E
Patients with paraneoplastic pemphigus has painful oral lesions and polymorphous skin eruption. The patients die from complications of underlying malignancy. Bronchiolitis obliterans has also been reported as a complication and cause of death.
203 -Which of the following is NOT associated with this disease of symmetric induration caused mucin deposition?
A. Diabetes mellitus
B. Streptococcal infection
C. Monoclonal gammopathy
D. Hepatitis C
E. All of the answers are associated with this disease
►D
Scleredema is a type of dermal degenerating mucinosis characterized by diffuse symmetric induration of the upper body. 3 types of scleredema have been described. The first type is seen in children following a stretococcal infection. The second type is associated with a monoclonal gammopathy. The third type is related to insulin dependent diabetes.
204- Reiter’s syndrome is known as reactive arthritis and involves conjunctivitis, urethritis and arthritis after a urogenital or gastrointestinal infection. The gene associated with Reiter’s syndrome is:
A. HLA-B27
B. HLA-B32
C. HLA-DQ23
D. HLA-B29
E. HLA-DQ34
►A
Reiter’s syndrome is associated with HLA-B27 and can have a variable course often 3-12 months. There is a chronic sequelae in 15-20% of patients.
205- Patients that are diagnosed with Grover’s disease have exacerbation most commonly in the:
A. Winter
B. Fall
C. Summer
D. Spring
E. Humidity
►A
Patients diagnosed with Grover’s disease have exacerbation commonly in the winter time. Authors suggest that there may be due to xerotic epidermis and impaired epidermal integrity and decreased sweat production. It is not caused by sweating and heat and is not increased in summer.
206- Which of the following is a feature of Sneddon’s syndrome?
A. Venous thromboses
B. Hepatosplenomegaly
C. anti-Scl-70 autoantibodies
D. M. tuberculosis infection
E. C2 deficiency
►A
Sneddon’s syndrome presents as livedo reticularis and livedoid vasculitis associated with ischemic cerebrovascular lesions, hypertension, and extracerebral arterial and venous thromboses.
207 -The risk of developing systemic involvement in a patient with nodular amyloidosis is:
A. 1-4%
B. 5-10%
C. 25-30%
D. 50-60%
E. >95%
►B
Nodular amyloidosis is a rare type of primary cutaneous amyloidosis made up of AL protein. The risk for systemic involvement is 7%.
208- Almost all patients with SLE have positive ANAs. A patient can have ANA-negative SLE if they only make antibodies to:
A. ssDNA
B. Sm
C. U1RNP
D. Ro
E. dsDNA
►A
ANA-negative SLE is uncommon. The ANA has a high negative predictive value and low positive predictive value. ANA-negative SLE can result if a patient only makes antibodies to ssDNA, which is not detected by most tests.
209- Lichen planus pemphigoides has been associated with an antigen to which structure?
A. Type XVII collagen
B. Hemidesmosome plaque
C. Desmoglein 3
D. Desmoglein 1
E. Type VII collagen
►A
Type XVII collagen, or BPag2 (180 kD) has been associated with LP pemphigoides. The hemisdesomosome plauqe is also known as BPag1 (230 - kD), desmoglein 1 and 3 are associated with the Pemphigous family, and antibodies to Type VII collagen is found in Epidermolysis Bullosa Aquisita
210- First line therapy for subcorneal pustular dermatosis is:
A. Dapsone
B. Tetracycline
C. Vitamin E
D. Topical retinoids
E. Topical steroids
►A
First line therapy for subcorneal pustular dermatosis or Sneddon-Wilkinson disease is dapsone 50-200mg/day. Sulfapyridine, acitretin, PUVA, NB UVB, topical and systemic steroids, vitamin E and antibiotics have been reported to be helpful also.
211 -The most common association with the generalized lesions shown is:
A. Hodgkin’s disease
B. Non-Hodgkin’s lymphoma
C. Granulomatous mycosis fungoides
D. HIV infection
E. Diabetes mellitus
►E
In a large retrospective study of 1383 patients, diabetes mellitus was diagnosed in 21% of patients with generalized GA, compared with 9.7% of patients with localized GA. Othere choices also have been reported to be associated with granuloma annulare.
212- Angiolymphoid hyperplasia with eosinophilia most commonly occurs on the:
A. Ear
B. Lip
C. Dorsal hands
D. Trunk
E. Lower extremities
►A
Angiolymphoid hyperplasia with eosinophilia often presents with red to reddish-brown papules or nodules on the head or neck. While lesions can occur on any body site, most common locations are the ears and scalp.
213- Xanthoma striata palmaris are diagnostic of:
A. Familial hypertriglyceridemia (type IV)
B. Familial dysbetalipoproteinemia (type III)
C. Familial lipoprotein lipase deficiency (AR)
D. Familial lipoprotein lipase deficiency (AD)
E. Apoprotein CII deficiency
►B
Xanthoma striata palmaris are diagnostic of type III dysbetalipoproteinemia (AR; broad beta disease). This condition also presents with palmar, planar, tendinous, tuberous, eruptive, and intertriginous xanthomas, increased IDL, and atherosclerosis. It is associated with diabetes, gout, and obesity.
214 -Relapsing polychondritis is characterized by circulating auto-antibodies to which of the following?
A. Collagen type IV
B. Collagen type VII
C. Collagen type II
D. Elastin
E. Chondroitin sulfate
►C
Relapsing polychondritis is a chronic, recurrent rheumatic disease characterized by cartilaginous inflammation involving the nose, ears, and trachea. A non-erosive inflammatory arthritis may also be present, and affected patients may suffer from neurosensory hearing loss, tinnitus, and/or vertigo secondary to vestibular or cochlear damage. Patients may have circulating autoantibodies to type II collagen (found exclusively in cartilage) with circulating titers corresponding to disease activity.
215- Which of the following medications is NOT associated with exacerbating or causing this eruption?
A. Lithium
B. Prednisone
C. Phenytoin
D. Isoniazid
E. Propranolol
►E
Acne and acneiform eruptions can be caused or exacerbated by cortiosteroids, oral contraceptives, androgens, ACTH, lithium, phenytoin, halogens, INH, and haloperidol.
216 -What is the treatment of choice for neurotic excoriations?
A. Risperidone
B. Doxepin
C. Olanzapine
D. Gabapentin
E. Diphenhydramine
►B
Doxepin is the treatment of choice for patients with neurotic excoriations. It has both antidepressant and antipruritic effects. Combined psychiatric and pharmacologic intervention is recommended. Other agents such as risperidone, olanzapine, gabapentin, and diphenhydramine are not drugs of choice for this disorder.
217 -Diarrhea, Dementia and a photosensitive dermatitis are associated with a deficiency of which vitamin?
A. Niacin
B. Biotin
C. Thiamine
D. Riboflavin
E. Pyridoxine
►A
Niacin (Vitamin B3) deficiency is associated with a photosensitive dermatitis, diarrhea and dementia. The photosensitive dermatitis classically involves the face, neck and upper chest (Casal’s necklace) and forearms (as pictured). Other findings may include angular cheilitis and thickening and hyperpigmentation of skin overlying bony prominences.
218 -What complication can occur if lichen sclerosis is left untreated?
A. Development of a squamous cell carcinoma
B. Testicular torsion
C. Erectile dysfunction
D. Impotence
E. Pseudo-ainhum
►A
Lichen sclerosis is most commonly located in the genitalia. Longstanding lichen sclerosis is thought to be a risk factor for the development of a squamous cell carcinoma.
219 -Concomitant discoid lupus is found in what percentage of patients with subacute cutaneous lupus?
A. 5%
B. 20%
C. 40%
D. 60%
E. 80%
►B
Sub acute cutaneous lupus presents with scaly papules that evolve into either psoriasiform or polycyclic annular lesions (more commonly). Sun-exposed surfaces of the face and neck are commonly involves. Photo sensitivity is seen in 40%, the hard palate is involved in 40%, and concomitant DLE is seen in 20%.
220-What is the most common digital manifestation of patients with systemic lupus erythematosus?
A. Periungual telangiectasia
B. Raynaud phenomenon
C. Splinter hemorrhages
D. Sclerodactyly
E. Digital pitting scars
►B
This is the most common digital manifestation in patients with SLE and it could present in up to 30% of the patients. It is a paroxysmal vasospam of digits in response to cold exposure or emotional stress.
221 -What is the condition which is a diagnostic cutaneous manifestation of sarcoid?
A. Discoid lupus
B. Lupus pernio
C. Lupus vulgaris
D. Granulomatous rosacea
E. Rhinoscleroma
►B
Sarcoidosis is a systemic disease charcterized by non-caseating granulomas. Organ involved include the skin, lung, liver and eyes. Lupus pernio manifests as indurated, red-brown, swollen plaques of the nose, lips, cheeks and ears.
222 -What is the most likely diagnosis?
A. Dermatofibromasarcoma protuberans
B. Dermatofibroma
C. Lobomycosis
D. Blastomycosis
E. Keloid
►E
Keloids are dense overgrowths of fibrous tissue that generally form a response to tissue injury. Lobomycosis and dermatofibromasarcoma protuberans may mimic keloids.
223- A 30 year old pregnant female presents with a lesion that has rapidly appeared. She is in her third trimester with normal pregnancy course to date. What is the most likely diagnosis?
A. Hemangioma
B. Glomus tumor
C. Bacillary angiomatosis
D. Pyogenic granuloma
E. Inflammed wart
►D
It is one of several vascular lesions that can present on the skin during pregnancy. Other common vascular lesions include palmar erythema, spider angiomas, varicose veins, cavernous hemangiomas, glomus tumors, and hemorrhoids. Etiologic factors in development of vascular lesions during pregnancy thought to be a combination of increased hormones and increased intravascular pressure. Pyogenic granulomas may develop as a reaction to trauma. They often regress after delivery and treatment is not usually required unless they cause excessive bleeding.
224- Nekam’s Disease:
A. Is generally responsive to topical and intralesional steroids
B. Characteristically lacks scale
C. Rarely involves the buttocks
D. Presents with a reticulate pattern on the dorsal hands and feet
E. Presents with hypopigmented, atrophic lesions on the extremities
►D
Nekam’s Disease (keratosis lichenoides chronica) presents with violaceous papules and nodules, hyperpigmented and hyperkeratotic, covered with gray scales. There is often a linear and reticulate pattern on the dorsal hands and feet, extremities and buttocks. This condition is generally very refractory to treatment.
225- Acquired angioedema is characterized by:
A. Inheritance
B. Normal levels of C1
C. Self-limited course
D. Association with underlying malignancy
E. Increased C1 esterase inhibitor
►D
Hereditary angioedema is an autosomal dominant condition associated with recurrent attacks of angioedema of the skin, respiratory, and gastrointestinal tract, without urticaria and is caused by a functional deficiency in C1 esterase inhibitor. In contrast, acquired angioedema is caused by a depletion in C1 esterase inhibitor. There are two forms of acquired angioedema; one results from the production of antibodies directed against C1 esterase inhibitor, and the other is seen in association with underlying malignancies, especially B cell lymphomas, with antibody production against overexpressed paraproteins. Acquired angioedema can be distinguished from hereditary angioedema by differences in serum levels of C1. While the level is markedly decreased in acquired angioedema, a normal C1 is seen in patients with hereditary angioedema.
226- This syndrome is a variant of Dowling-Degos disease with reticulated hyperpigmentation on the flexures and scaly erythematous papules on flexures on the trunk, AD, keratin 5 mutation. This syndrome is called:
A. Galli-Galli Syndrome
B. Dowling Degos type I
C. Haim-Munk Syndrome
D. Dengue syndrome
E. Dowling Degos type II
►A
Gali-gali syndrome is a variant of Dowling Degos syndrome. It is characteristic of 1- to 2-mm slightly keratotic red to dark brown papules which are focally confluent in a reticulate pattern. The disease is also characterized by slowly progressive and disfiguring reticulate hyperpigmentation of the flexures.
227- Acquired C1 esterase inhibitor deficiency results in:
A. A normal level of serum C1q
B. A decreased level of serum C1q
C. None of these answers are correct
D. A presentation of angioedema very early in life
E. A normal level of C4
►B
Acquired C1 esterase inhibitor deficiency occurs in the setting of lymphoproliferative disease, monoclonal gammopathy of undetermined significance, or rheumatologic disease and results in a decreased serum C1q level. Inherited C1 esterase inhibitor deficiency shows an autosomal dominant inheritance, and earlier presentation, and a normal level of serum C1q. In the inherited disease, the C1 esterase inhibitor may display normal levels but be functionally impaired.
228- The most common systemic manifestations of systemic sclerosis are:
A. Gastrointestinal
B. Cardiovascular
C. Pulmonary
D. Renal
E. Neurologic
►A
Esophageal dysfunction is the most systemic finding in systemic sclerosis (>90%). Dysphagia results from decreased peristalsis and may occur before skin findings are seen. Small intestinal involvement is also common. Pulmonary fibrosis, myocardial fibrosis (seen in 50-70%), cardiac conduction defects, heart failure, pericarditis with effusion, and renal disease with slowly progressive uremia may all be seen.
229- The joint most frequently affected in acne fulminans is the:
A. Elbow
B. Intervertebral
C. Distal interphalangeal joints of the hand
D. Sternoclavicular
E. Sacroiliac
►D
Acne fulminans may be associated with lytic bone changes which are indicative of a sterile osteomyelitis. The sternoclavicular and chest wall joints are most frequently affected.
230 -A person that always burns and sometimes tans has the skin phototype:
A. II
B. I
C. III
D. IV
E. V
►A
A person that always burns and sometimes tans is type II skin.
231- What is the most common malignancy associated with dermatomyositis in this female?
A. Brain cancer
B. Thyroid cancer
C. Rectal cancer
D. Ovarian cancer
E. Liver cancer
►D
Dermatomyositis is an autoimmune polymyositis with characteristic cutaneous findings. Patients with dermatomyositis should be screened for a underlying malignancy. Risk factors for having an underlying malignancy include a negative ANA, adult age, and female gender. Ovarian cancer is one of the most frequently associated with dermatomyositis.
232- A 40 year-old man presents with a complaint of nail changes for several years. He was treated by an outside physician with terbinafine without improvement. On further questioning, he reports morning shoulder stiffness and back pain that lasts 1-2 hours and improves with activity. Which of the following is true regarding his condition?
A. 50% of patients present with joint disease prior to skin involvement
B. Bony erosions are not commonly seen on radiographs
C. Cyclosporine is effective in treating the arthritis in this condition
D. A positive rheumatoid factor may be seen
E. Joint disease correlates with severity of skin involvement
►D
This patient has psoriatic nail changes and a history suggestive of psoriatic arthritis. Psoriatic arthritis is an inflammatory arthropathy associated with psoriasis. In 80% of patients the rheumatoid factor is negative; however a positive rheumatoid factor may sometimes be seen. 80% of patients present with skin disease first. Large eccentric erosions are classically present on radiographs. Cyclosporine is not effective in treating psoriatic arthritis. Mild skin disease may be associated with moderate-to-severe joint disease, and vice versa.
233- Which enzyme can be a useful adjunct test to diagnose zinc deficiency where the serum zinc level is normal or near normal?
A. Zinc sulfatase
B. Alkaline phosphatase
C. AST
D. ALT
E. Creatinine kinase
►B
A low serum alkaline phosphatase, a zinc dependent enzyme, may be a valuable adjuctive test where the serum zinc level is normal or near normal. The diagnosis of zinc deficiency should be suspected in at-risk individuals with acral or periorificial dermatitis. It is usually confirmed by low serum zinc levels.
234- Which antibody is 93% specific for Sjogren’s syndrome?
A. anti-Ro
B. anti-La
C. anti-fodrin
D. anti-Schirmer
E. RF
►C
Sjogren’s syndrome is a triad of keratoconjunctivitis sicca, xerostomia, and rheumatoid arthritis. More than 90% of patients are women. Labial salivary gland biopsy is useful for diagnosis, and the Schirmer test for xerostomia detects diminished glandular secretions. Laboratory findings often include positive cryoglobulins, anti-Ro, anti-La, and RF positivity. Antibodies to fodrin are 93% specific for this diagnosis.
235- Elastophagocytosis is characteristic of which condition?
A. Sarcoidosis
B. Granuloma annulare
C. Necrobiosis lipoidica
D. Annular elastolytic giant cell granuloma
E. Palisading neutrophilic and granulomatous dermatitis
►D
Annular elastolytic giant cell granuloma is characterized by asymptomatic annular plaques on sun exposed surfaces. Elastic fibers are absent in the center of lesions. The disease is seen primarily in middle aged women.